Bullets

HI there....welcome To test visual acuity, the nurse should ask the patient to cover each eye separately and read the eye chart with glasses and without, as appropriate. Fundamental of Nursing Before teaching any procedure to the patient, the nurse must first assess the patient’s willingness to learn and his current knowledge. Fundamentals of Nursing A blood pressure cuff that is too narrow can cause a falsely elevated blood pressure reading. Fundamentals of Nursing When preparing a single injection for a patient who takes regular and NPH insulin, the nurse should draw the regular insulin into the syringe first because it is clear and can be measured more accurately than the NPH insulin, which is turbid. Fundamentals of Nursing Rhonchi refers to the rumbling sounds heard on lung auscultation; they are more pronounced during expiration than during inspiration. Fundamentals of Nursing Gavage refers to forced feeding, usually through a gastric tube (a tube passed into the stomach by way of the mouth). Fundamentals of Nursing According to Maslow’s hierarchy of needs, physiologic needs (air, water, food, shelter, sex, activity, and comfort) have the highest priority. Fundamentals of Nursing Checking the identification band on a patient’s wrist is the safest and surest way to verify a patient’s identity. Fundamentals of Nursing A patient’s safety is the priority concern in developing a therapeutic environment. Fundamentals of Nursing The nurse should place the patient with a Sengstaken-Blakemore tube in semi-Fowler’s position. Fundamentals of Nursing The nurse can elicit Trousseau’s sign by occluding the brachial or radial artery; hand and finger spasms during occlusion indicate Trousseau’s sign and suggest hypocalcemia. Fundamentals of Nursing For blood transfusion in an adult, the appropriate needle size is 16 to 20G. Fundamentals of Nursing Pain that incapacities a patient and can’t be relieved by drugs is called intractable pain. Fundamentals of Nursing In an emergency, consent for treatment can be obtained by fax, telephone, or other telegraphic transmission. Fundamentals of Nursing Decibel is the unit of measurement of sound. Fundamentals of Nursing Informed consent is required for any invasive procedure. Fundamentals of Nursing A patient who can’t write his or her name to give consent for treatment must have his or her X witnessed by two persons, such as a nurse, priest, or doctor. Fundamentals of Nursing The Z-track I.M. injection technique seals medication deep into the muscle, thereby minimizing skin irritation and staining. It requires a needle that is 1’’ (2.5 cm) or longer. Fundamentals of Nursing A registered nurse (RN) should assign a licensed vocational nurse (LVN) or licensed practical nurse (LPN) to perform bedside care, such as suctioning and medication administration. Fundamentals of Nursing The therapeutic purposed of a mist tent is to increase hydration of secretions. Fundamentals of Nursing If a patient can’t void, the first nursing action should be bladder palpation to assess for bladder distention. Fundamentals of Nursing The patient who uses a cane should carry it on the unaffected side and advance it at the same time as the affected extremity. Fundamentals of Nursing To fit a supine patient for crutches, the nurse should measure from the axilla to the sole and add 2” (5 cm) to that measurement. Fundamentals of Nursing Assessment begins with the nurse’s first encounter with the patient and continues throughout the patient’s stay. The nurse obtains assessment data through the health history, physical examination, and review of diagnostic studies. Fundamentals of Nursing The appropriate needle size for an insulin injection is 25G and ⅝" (1.5 cm) long. Fundamentals of Nursing Residual urine refers to urine that remains in the bladder after voiding. The amount of residual urine normally ranges from 50 to 100 ml. Fundamentals of Nursing The five stages of the nursing process are assessment, nursing diagnosis, planning, implementation, and evaluation. Fundamentals of Nursing Planning refers to the stage of the nursing process in which the nurse assigns priorities to nursing diagnoses, defines short-term and long-term goals and expected outcomes, and establishes the nursing care plan. Fundamentals of Nursing Implementation refers to the stage of the nursing process in which the nurse puts the nursing care plan into action, delegates specific nursing interventions to members of the nursing team, and charts patient responses to nursing interventions. Fundamentals of Nursing Evaluation refers to the stage of the nursing process in which the nurse compares objective and subjective data with the outcome criteria and, if needed, modifies the nursing care plan, making the nursing process circular. Fundamentals of Nursing In the event of fire, the nurse should (1) remove the patient, (2) call the fire department, (3) attempt to contain the fire by closing the door, and (4) extinguish the fire, if it can be done safely. Fundamentals of Nursing Before administering any as need pain medication, the nurse should ask the patient to indicate the pain’s location. Fundamentals of Nursing Jehovah’s Witnesses believe that they shouldn’t receive blood components donated by other people. Fundamentals of Nursing When providing oral care for an unconscious patient, the nurse should position the patient on the side to minimize the risk of aspiration. Fundamentals of Nursing During assessment of distance vision, the patient should stand 20’ (6.1 m) from the chart. Fundamentals of Nursing The ideal room temperature for a geriatric patient or one who is extremely ill ranges form 66º to 76º F (18.8º to 24.4º C). Fundamentals of Nursing Normal room humidity ranges from 30% to 60%. Fundamentals of Nursing Hand washing is the single best method of limiting the spread of microorganisms. Hands should be washed for 10 seconds after routine contact with a patient and after gloves are removed. Fundamentals of Nursing To catheterize a female patient, the nurse should place her in the dorsal recumbent position. Fundamentals of Nursing A positive Homan’s sign may indicate thrombophlebitis. Fundamentals of Nursing Electrolytes in a solution are measured in milliequivalents per liter (mEq/L). A milliequivalent equals the number of milligrams per 100 milliliters of a solution. Fundamentals of Nursing Metabolism takes a place in two phases: anabolism (the constructive phase) and catabolism (the destructive phase). Fundamentals of Nursing The basal metabolic rate represents the amount of energy needed to maintain essential body functions. It is measured when the patient is awake and resting, hasn’t eaten for 14 to 18 hours, and is in a comfortable, warm environment. Fundamentals of Nursing Dietary fiber (roughage), which is derived from cellulose, supplies bulk, maintains adequate intestinal motility, and helps establish regular bowel habits. Fundamentals of Nursing Alcohol is metabolized primarily in the liver. Smaller amounts are metabolized by the kidneys and lungs. Fundamentals of Nursing Petechiae refers to tiny, round, purplish red spots that appear on the skin and mucous membranes as a result of intradermal or submucosal hemorrhage. Fundamentals of Nursing Purpura refers to a purple skin discoloration caused by blood extravasation. Fundamentals of Nursing Glucose-6-phosphate dehydrogenase (C6PD) deficiency is an inherited metabolic disorder characterized by red blood cells that are deficient in G6PD, a critical enzyme in aerobic glycolysis. Fundamentals of Nursing According to the standard precautions recommended by the Centers for Disease Control and Prevention, the nurse shouldn’t recap needles after use because most needle sticks result from missed needle recapping. Fundamentals of Nursing The nurse administers a drug by I.V. push by delivering the dose directly into a vein, I.V. tubing, or catheter with a needle and syringe. Fundamentals of Nursing When changing the ties on a tracheostomy tube, the nurse should leave the old ties in place until the new ones are applied. Fundamentals of Nursing A nurse should have assistance when changing the ties on a tracheostomy tube. Fundamentals of Nursing A filter is always used for blood transfusions. Fundamentals of Nursing A four-point (quad) cane is indicated when a patient needs more stability than a regular cane can provide. Fundamentals of Nursing The patient should carry a cane on the unaffected side to promote a reciprocal gait pattern and distribute weight away from the affected leg. Fundamentals of Nursing A good way to begin a patient interview is to ask “What made you seek medical help?” Fundamentals of Nursing The nurse should adhere to standard precautions for blood and body fluids when caring for all patients. Fundamentals of Nursing Potassium (K+) is the most abundant cation in intracellular fluid. Fundamentals of Nursing In the four-point gait (or alternating gait), the patient first moves the right crutch followed by the left foot and then the left crutch followed by the right foot. Fundamentals of Nursing In the three-point gait, the patient moves two crutches and the affected leg simultaneously and then moves the unaffected leg. Fundamentals of Nursing In the two-point gait, the patient moves the right leg and the left crutch simultaneously and then moves the left leg and the right crutch. Fundamentals of Nursing Vitamin B complex, the water-soluble vitamins essential for metabolism, include thiamine (B1), riboflavin (B2), niacin (B3), pyridoxine (B6), and cyanocobalamin (B12). Fundamentals of Nursing When being weighed, an adult patient should be lightly dressed and shoeless. Fundamentals of Nursing Before taking an adult’s oral temperature, the nurse should ensure that the patient hasn’t smoked or consumed hot or cold substances in the past 15 minutes. Fundamentals of Nursing The nurse shouldn’t take a rectal temperature on an adult patient if the patient has a cardiac disorder; anal lesions, or bleeding hemorrhoids or has recently undergone rectal surgery. Fundamentals of Nursing In a patient with cardiac problems, rectal temperature measurement may stimulate a vagal response, leading to vasodilation and decreased cardiac output. Fundamentals of Nursing When recording pulse amplitude and rhythm, the nurse should use these descriptive measures: +3 indicates a bounding pulse (readily palpable and forceful); +2, a normal pulse (easily palpable); +1, a thready or weak pulse (difficult to detect); and 0, an absent pulse (not detectable). Fundamentals of Nursing The intraoperative period begins when a patient is transferred to the operating room bed and ends when the patient is admitted to the postanesthesia recovery unit. Fundamentals of Nursing On the morning of surgery, the nurse should ensure that the informed consent form has been signed; that the patient hasn’t taken anything by mouth since midnight, has taken a shower with antimicrobial soap, has had mouth care (without swallowing the water, has removed common jewelry, and has received preoperative medication as prescribed; and that vital signs have taken and recorded. Artificial limbs and other prostheses are usually removed. Fundamentals of Nursing Comfort measures, such as positioning the patient, performing backrubs, and providing a restful environment, may decrease the patient’s need for analgesics or may enhance their effectiveness. Fundamentals of Nursing A drug has three names: its generic name, which is used in official publications; its trade name or brand name (such as Tylenol), which is selected by the drug company; and its chemical name, which describes the drug’s chemical composition. Fundamentals of Nursing The patient should take a liquid iron preparation through a straw to avoid staining the teeth. Fundamentals of Nursing The nurse should use the Z-track method to administer an I.M. injection of iron dextran (Imferon). Fundamentals of Nursing An organism may enter the body through the nose, mouth, rectum, urinary or reproductive tract, or skin. Fundamentals of Nursing In descending order, the levels of consciousness are alertness, lethargy, stupor, light coma, and deep coma. Fundamentals of Nursing To turn a patient by logrolling, the nurse folds the patient’s arms across the chest; extends the patient’s legs and inserts a pillow between them, if indicated; places a draw sheet under the patient; and turns the patient by slowly and gently pulling on the draw sheet. Fundamentals of Nursing The diaphragm of the stethoscope is used to hear high-pitched sounds such as breath sounds. Fundamentals of Nursing A slight blood pressure difference (5 to 10 mm Hg) between right and left arms is normal. Fundamentals of Nursing The nurse should place the blood pressure cuff 1'' (2.5 cm) above the antecubital fossa. Fundamentals of Nursing When instilling ophthalmic ointments, waste the first bed of ointment and then apply from the inner canthus to the outer canthus; twist the medication tube to detach the ointment. Fundamentals of Nursing The nurse should use a leg cuff to measure blood pressure in an obese patient. Fundamentals of Nursing If the blood pressure cuff is applied too loosely, the reading will be falsely elevated. Fundamentals of Nursing Ptosis refers to eyelid drooping. Fundamentals of Nursing A tilt table is useful for a patient with a spinal cord injury, orthostatic hypotension, or brain damage because it can move the patient gradually from a horizontal to a vertical (upright) position. Fundamentals of Nursing To perform venipuncture with the least injury to the vessel, the nurse should turn the bevel upward when the vessel’s lumen is larger than the needle and turn it downward when the lumen is only slightly larger than the needle. Fundamentals of Nursing To move the patient to the edge of the bed for transfer, follow these steps: (1) Move the patient’s head and shoulders toward the edge of the bed. (2) Move the patient’s feet and legs to the edge of the bed (crescent position). (3) Place both the arms well under the patient’s hips and straighten the back while moving the patient toward the edge of the bed. Fundamentals of Nursing When being measured for crutches, a patient should wear his or her shoes. Fundamentals of Nursing The nurse should attach a restraint to a part of the bed frame that moves with the head, not to the mattress or side rails. Fundamentals of Nursing The mist in a mist tent should never become so dense that it obscures clear visualization of the patient’s respiratory pattern. Fundamentals of Nursing To administer heparin subcutaneously, the nurse should follow these steps: (1) Clean, but don’t rub, the site with alcohol. (2) Stretch the skin taut or pick up a well-defined skin fold. (3)Hold the shaft of the needle in a dart position. (4)Insert the needle into the skin at a right (90-degree) angle. (5)Firmly depress the plunger; but don’t aspirate. (6)Leave the needle in place for 10 seconds. (7)Withdraw the needle gently at the same angle it was inserted. (8)Apply pressure to the injection site with an alcohol pad. Fundamentals of Nursing For a sigmoidoscopy, the nurse should place the patient in a knee-chest or Sims’ position, depending on the doctor’s preference. Fundamentals of Nursing Maslow’s hierarchy of needs must be met in the following order: physiologic (oxygen, food, water, sex, rest, and comfort) safety and security, love and belonging, self-esteem and recognition, and self-actualization. Fundamentals of Nursing When caring for patient with a nasogastric tube, the nurse should apply a water-soluble lubricant to the nostril to prevent soreness. Fundamentals of Nursing During gastric lavage, a nasogastric tube is inserted, the stomach is flushed, and ingested substances are removed through the tube. Fundamentals of Nursing In documenting drainage on a surgical dressing, the nurse should include the size, color, and consistency of the drainage, for example, “10 mm of brown mucoid drainage noted on dressing.” Fundamentals of Nursing To elicit Babinski’s reflex, the nurse strokes the sole of the patient’s foot with a moderately sharp object, such as thumbnail. Fundamentals of Nursing In a positive Babinski’s reflex, the great toe dorsiflexes and the other toes fan out. Fundamentals of Nursing When assessing a patient for bladder distention, the nurse should check the contour of the lower abdomen for a rounded mass above the symphysis pubis. Fundamentals of Nursing The best way to prevent pressure ulcers is to reposition the bedridden patient at least every 2 hours. Fundamentals of Nursing Antiembolism stockings decompress the superficial blood vessels, thereby reducing the risk of thrombus formation. Fundamentals of Nursing The most convenient veins for venipuncture in a adult patient are the basilic and median cubital veins in the antecubital space. Fundamentals of Nursing From 2 to 3 hours before beginning a tube feeding, the nurse should aspirate the patient’s stomach contents to verify adequate gastric emptying. Fundamentals of Nursing People with type O blood are considered to be universal donors. Fundamentals of Nursing People with type AB blood are considered to be universal recipients. Fundamentals of Nursing Herts (Hz) refers to the unit of measurement of sound frequency. Fundamentals of Nursing Hearing protection is required when the sound intensity exceeds 84 dB; double hearing protection is required if it exceeds 104 dB. Fundamentals of Nursing Prothrombin, a clotting factor, is produced in the liver. Fundamentals of Nursing If a patient is menstruating when a urine sample is collected, the nurse should note this on the laboratory slip. Fundamentals of Nursing During lumbar puncture, the nurse must note the initial intracranial pressure and the cerebrospinal fluid color. Fundamentals of Nursing A patient who can’t cough to provide a sputum sample for culture may require a heated aerosol treatment to facilitate removal of a sample. Fundamentals of Nursing If eye ointment and eyedrops must be instilled in the same eye, the eyedrops should be instilled first. Fundamentals of Nursing When leaving an isolation room, the nurse should remove the gloves before the mask because fewer pathogens are on the mask. Fundamentals of Nursing Skeletal traction is applied to a bone using wire pins or tons. It is the most effective means of traction. Fundamentals of Nursing The total parenteral nutrition solution should be stored in a refrigerator and removed 30 to 60 minutes before use because delivery of a chilled solution can cause pain, hypothermia, venous spasm, and venous constriction. Fundamentals of Nursing Medication isn’t routinely injected I.M. into edematous tissue because it may not be absorbed. Fundamentals of Nursing When caring for a comatose patient, the nurse should explain each action to the patient in a normal voice. Fundamentals of Nursing When cleaning dentures, the sink should be lined with a washcloth. Fundamentals of Nursing A patient should void within 8 hours after surgery. Fundamentals of Nursing An EEG identifies normal and abnormal brain waves. Fundamentals of Nursing Stool samples for ova and parasite tests should be delivered to the laboratory without delay or refrigeration. Fundamentals of Nursing The autonomic nervous system regulates the cardiovascular and respiratory systems. Fundamentals of Nursing When providing tracheostomy care, the nurse should insert the catheter gently into the tracheostomy tube. When withdrawing the catheter, the nurse should apply intermittent suction for no more than 15 seconds and use a slight twisting motion. Fundamentals of Nursing A low-residue diet includes such as foods as roasted chicken, rice, and pasta. Fundamentals of Nursing A rectal tube should not be inserted for longer than 20 minutes; it can irritate the mucosa of the rectum and cause a loss of sphincter control. Fundamentals of Nursing A patient’s bed bath should proceed in this order: face, neck, arms, hands, chest, abdomen, back, legs, perineum. Fundamentals of Nursing When lifting and moving a patient, the nurse should use the upper leg muscles most to prevent injury. Fundamentals of Nursing Patient preparation for cholecystography includes ingestion of a contrast medium and a low-fat evening meal. Fundamentals of Nursing During occupied bed changes, the patient should be covered with a black blanket to promote warmth and prevent exposure. Fundamentals of Nursing Anticipatory grief refers to mourning that occurs for an extended time when one realizes that death is inevitable. Fundamentals of Nursing The following foods can alter stool color: beets (red), cocoa (dark red or brown), licorice (black), spinach (green), and meat protein (dark brown). Fundamentals of Nursing When preparing a patient for a skull X-ray, have the patient remove all jewelry and dentures. Fundamentals of Nursing The fight-or-flight response is a sympathetic nervous system response. Fundamentals of Nursing Bronchovesicular breath sounds in peripheral lung fields are abnormal and suggest pneumonia. Fundamentals of Nursing Wheezing refers to an abnormal, high-pitched breath sound that is accentuated on expiration. Fundamentals of Nursing Wax or a foreign body in the ear should be gently flushed out by irrigation with warm saline solution. Fundamentals of Nursing If a patient complains that his hearing aid is “not working,” the nurse should check the switch first to see if it’s turned on and then check the batteries. Fundamentals of Nursing The nurse should grade hyperactive biceps and triceps reflexes +4. Fundamentals of Nursing If two eye medications are prescribed for twice-daily instillation, they should be administered 5 minutes apart. Fundamentals of Nursing In a postoperative patient, forcing fluids helps prevent constipation. Fundamentals of Nursing The nurse must administer care in accordance with standards of care established by the American Nurses Association, state regulations, and facility policy. Fundamentals of Nursing The kilocalorie (kcal) is a unit of energy measurement that represents the amount of heat needed to raise the temperature of 1 kilogram of water 1º C. Fundamentals of Nursing As nutrients move through the body, they undergo ingestion, digestion, absorption, transport, cell metabolism, and excretion. Fundamentals of Nursing The body metabolizes alcohol at a fixed rate regardless of serum concentration. Fundamentals of Nursing In an alcoholic beverage, its proof reflects its percentage of alcohol multiplied by 2. For example, a 100-proof beverage contains 50% alcohol. Fundamentals of Nursing A living will is a witnessed document that states a patient’s desire for certain types of care and treatment, which depends on the patient’s wishes and views and quality of life. Fundamentals of Nursing The nurse should flush a peripheral heparin lock every 8 hours (if it wasn’t used during the previous 8 hours) and as needed with normal saline solution to maintain patency. Fundamentals of Nursing Quality assurance is a method of determining whether nursing actions and practices meet established standards. Fundamentals of Nursing The five rights of medication administration are the right patient, right medication, right dose, right route of administration, and the right time. Fundamentals of Nursing Outside of the hospital setting, only the sublingual and transligual forms of nitroglycerin should be used to relieve acute anginal attacks. Fundamentals of Nursing The implementation phase of the nursing process involves recording the patient’s response to the nursing plan, putting the nursing plan into action, delegating specific nursing interventions, and coordinating the patient’s activities. Fundamentals of Nursing The Patient’s Bill of Rights offers guidance and protection to patients by stating the responsibilities of the hospital and its staff toward patients and their families during hospitalization. Fundamentals of Nursing To minimize the omissions and distortion of facts, the nurse should record information as soon as it is gathered. Fundamentals of Nursing When assessing a patient’s health history, the nurse should record the current illness chronologically, beginning with the onset of the problem and continuing to the present. Fundamentals of Nursing Drug administration is a dependent activity. The nurse can administer or withhold a drug only with the doctor’s permission. Fundamentals of Nursing The nurse shouldn’t give false assurance to a patient. Fundamentals of Nursing After receiving preoperative medication, a patient isn’t competent to sign an informed consent form. Fundamentals of Nursing When lifting a patient, a nurse uses the weight of her body instead of the strength in her arms. Fundamentals of Nursing A nurse may clarify a doctor’s explanation to a patient about an operation or a procedure but must refer questions about informed consent to the doctor. Fundamentals of Nursing The nurse shouldn’t use her thumb to take a patient’s pulse rate because the thumb has a pulse of its own and may be confused with the patient’s pulse. Fundamentals of Nursing An inspiration and an expiration count as one respiration. Fundamentals of Nursing Normal respirations are known as eupnea. Fundamentals of Nursing During a blood pressure measurement, the patient should rest the arm against a surface because using muscle strength to hold up the arm may raise the blood pressure. Fundamentals of Nursing Major unalterable risk factors for coronary artery disease include heredity, sex, race, and age. Fundamentals of Nursing Inspection is the most frequently used assessment technique. Fundamentals of Nursing Family members of an elderly person in a long-term care facility should transfer some personal items (such as photographs, a favorite chair, and knickknacks) to the person’s room to provide a homey atmosphere. Fundamentals of Nursing The upper respiratory tract warms and humidifies inspired air and plays a role in taste, smell, and mastication. Fundamentals of Nursing Signs of accessory muscle use include shoulder elevation, intercostal muscle retraction, and scalene and sternocleidosmastoid muscle use during respiration. Fundamentals of Nursing When patients use axillary crutches, their palms should bear the brunt of the weight. Fundamentals of Nursing Activities of daily living include eating, bathing, dressing, grooming, toileting, and interacting socially. Fundamentals of Nursing Normal gait has two phases: the stance phase, in which the patient’s foot rests on the ground, and the swing phase, in which that patient’s foot moves forward. Fundamentals of Nursing The phases of mitosis are prophase, metaphase, anaphase, and telophase. Fundamentals of Nursing The nurse should follow standard precautions in the routine care of all patients. Fundamentals of Nursing The nurse should use the bell of the stethoscope to listen for venous hums and cardiac murmurs. Fundamentals of Nursing The nurse can assess a patient’s general knowledge by asking questions such as “Who is the president for the United States?” Fundamentals of Nursing Cold packs are applied for the first 20 to 48 hours after an injury; then heat is applied. During cold application, the pack is applied for 20 minutes and then removed for 10 to 15 minutes to prevent reflex dilation (rebound phenomenon) and frostbite injury. Fundamentals of Nursing The pons is located above the medulla and consists of white matter (sensory and motor tracts) and gray matter (reflex centers). Fundamentals of Nursing The autonomic nervous system controls the smooth muscles. Fundamentals of Nursing A correctly written patient goal expresses the desired patient behavior, criteria for measurement, time frame for achievement, and conditions under which the behavior will occur. It is developed in collaboration with the patient. Fundamentals of Nursing The optic disk is yellowish pink and circular with a distinct border. Fundamentals of Nursing A primary disability results from a pathologic process; a secondary disability, from inactivity. Nurses usually are held liable for failing to keep an accurate count of sponges and other devices during surgery. Fundamentals of Nursing The best dietary sources of vitamin B6 are liver, kidney, pork, soybeans, corn, and whole-grain cereals. Fundamentals of Nursing Iron-rich foods, such as organ meats, nuts, legumes, dried fruit, leafy vegetables, eggs, and whole gains, generally have low water content. Fundamentals of Nursing Collaboration refers to joint communication and decision making between nurses and doctors designed to meet patients’ needs by integrating the care regimens of both professions in one comprehensive approach. Fundamentals of Nursing Bradycardia refers to a heart rate of fewer than 60 beats/minute. Fundamentals of Nursing A nursing diagnosis is a statement of a patient’s actual or potential health problems that can be resolved, diminished, or otherwise changed by nursing interventions. Fundamentals of Nursing During the assessment phase of the nursing process, the nurse collects and analyzes three types of data: health history, physical examination, and laboratory and diagnostic test data. Fundamentals of Nursing The patient’s health history consists primarily of subjective data, information supplied by patient. Fundamentals of Nursing The physical examination includes objective data obtained by inspection, palpation, percussion, and auscultation. Fundamentals of Nursing When documenting patient care, the nurse should write legibly, use only standard abbreviations, and sign every entry. The nurse should never destroy or attempt to obliterate documentation or leave vacant lines. Fundamentals of Nursing Factors that affect body temperature include time of day, age, physical activity, phase of menstrual cycle, and pregnancy. Fundamentals of Nursing The most accessible and commonly used artery for measuring a patient’s pulse rate is the radial artery, which is compressed against the radius to take the pulse rate. Fundamentals of Nursing The normal pulse rate of a resting adult is 60 to 100 beats/minute. The rate is slightly faster in women than in men and much faster in children than in adults. Fundamentals of Nursing Laboratory test results are an objective form of assessment data. Fundamentals of Nursing The measurement systems most often used in clinical practice are the metric system, apothecaries’ system, and household system. Fundamentals of Nursing Before signing an informed consent, a patient should know whether other treatment options are available and should understand what will occur during the preoperative, intraoperative, and postoperative phase; the risk involved; and the possible complications. The patient also should have a general idea of the time required from surgery to recovery and should have an opportunity to ask questions. Fundamentals of Nursing A patient must sign a separate informed consent form for each procedure. Fundamentals of Nursing During percussion, the nurse uses quick, sharp tapping of the fingers or hands against body surfaces to produce sounds (that helps determine the size, shape, position, and density of underlying organs and tissues), elicit tenderness, or assess reflexes. Fundamentals of Nursing Ballottement is a form of light palpation involving gentle, repetitive bouncing of tissues against the hand and feeling their rebound. Fundamentals of Nursing A foot cradle keeps bed linen off the patient’s feet, which prevent skin irritation and breakdown, especially in a patient with peripheral vascular disease or neuropathy. Fundamentals of Nursing If the patient is a married minor, permission to perform a procedure can be obtained form the patient’s spouse. Fundamentals of Nursing Gastric lavage is the flushing of the stomach and removal of ingested substances through a nasogastric tube. It can be used to treat poisoning or drug overdose. Fundamentals of Nursing During the evaluation step of the nursing process, the nurse assesses the patient’s response to therapy. Fundamentals of Nursing Bruits commonly indicate a life- or limb-threatening vascular disease. Fundamentals of Nursing O.U. means each eye; O.D., right eye; and O.S, left eye. Fundamentals of Nursing To remove a patient’s artificial eye, the nurse depresses the lower lid. Fundamentals of Nursing The nurse should use a warm saline solution to clean an artificial eye. Fundamentals of Nursing A thready pulse is very fine and scarcely perceptible. Fundamentals of Nursing Axillary temperature usually is 1º F lower than oral temperature. Fundamentals of Nursing After suctioning a tracheostomy tube, the nurse must document the color, amount, consistency, and odor of secretions. Fundamentals of Nursing On a medication prescription, the abbreviation p.c. means that the medication should be administered after meals. Fundamentals of Nursing After bladder irrigation, the nurse should document the amount, color, and clarity of the urine and the presence of clots or sediment. Fundamentals of Nursing Laws regarding patient self-determination vary from state to state. Therefore, the nurse must be familiar with the laws of the state in which she works. Fundamentals of Nursing Gauge refers to the inside diameter of a needle. The smaller the gauge, the larger the diameter. Fundamentals of Nursing An adult normally has 32 permanent teeth. Fundamentals of Nursing After turning a patient, the nurse should document the position used, time turned, and skin assessment findings. Fundamentals of Nursing PERRLA is an abbreviation for normal pupil assessment findings: pupils equal, round, and reactive to light with accommodation. Fundamentals of Nursing When purcussing a patient’s chest for postural drainage, the nurse’s hands should be cupped. Fundamentals of Nursing When measuring a patient’s pulse, the nurse should assess the rate, rhythms, quality, and strength. Fundamentals of Nursing Before transferring a patient from a bed to a wheelchair, the nurse should push the wheelchair’s footrests to the sides and lock its wheels. Fundamentals of Nursing When assessing respirations, the nurse should document the rate, rhythm, depth, and quality. Fundamentals of Nursing For a subcutaneous injection, the nurse should use a ⅝" 25G needle. Fundamentals of Nursing The notation “AA & O x 3” indicates that the patient is awake, alert, and oriented to person (knows who he is), place (knows where he is), and time (knows the date and time). Fundamentals of Nursing Fluid intake includes all fluids taken by mouth, including foods that are liquid at room temperature, such as gelatin, custard, and ice cream; I.V. fluids; and fluids administered in feeding tubes. Fluid output includes urine, vomitus, and drainage (such as from a nasogastric tube or from a wound) as well as blood loss, diarrhea or stool, and perspiration. Fundamentals of Nursing After administering an intradermal injection, the nurse shouldn’t massage the area because massage can irritate the site and interfere with results. Fundamentals of Nursing When administering an intradermal injection, the nurse should hold the syringe almost flat against the patient’s skin (at about a 15-degree angle) with the bevel up. Fundamentals of Nursing To obtain an accurate blood pressure, the nurse should inflate the manometer 20 to 30 mm Hg above the disappearance of the radial pulse before releasing the cuff pressure. Fundamentals of Nursing The nurse should count an irregular pulse for 1 full minute. Fundamentals of Nursing A patient who is vomiting while lying down should be placed in a lateral position to prevent aspiration of vomitus. Fundamentals of Nursing Prophylaxis is disease prevention. Fundamentals of Nursing Body alignment is achieved when the body parts are in proper relation to their natural position. Fundamentals of Nursing Trust is the foundation of a nurse-patient relationship. Fundamentals of Nursing Blood pressure in the force exerted by the circulating volume of blood on arterial walls. Fundamentals of Nursing Malpractice refers to the professional’s wrongful conduct, improper discharge of duties, or failure to meet standers of care, which causes harm to another. Fundamentals of Nursing As a general rule, nurses can’t refuse a patient care assignment; however, they may refuse to participate in abortions in most states. Fundamentals of Nursing A nurse can be found negligent if a patient is injured because the nurse failed to perform a duty that a reasonable and prudent person would perform or because the nurse performed an act that a reasonable and prudent person wouldn’t perform. Fundamentals of Nursing States have enacted Good Samaritan laws to encourage professionals to provide medical assistance at the scene of an accident without fear of a lawsuit arising from such assistance. These laws don’t apply to care provided in a health care facility. Fundamentals of Nursing A doctor should sign verbal and telephone orders within the time established by institutional policy, usually within 24 hours. Fundamentals of Nursing A competent adult has the right to refuse lifesaving medical treatment; however, the individual should be fully informed of the consequences of this refusal. Fundamentals of Nursing Although a patient’s health record or chart is the health care facility’s physical property, its contents belong to the patient. Fundamentals of Nursing Before a patient’s record can be released to a third party, the patient or patient’s legal guardian must give written consent. Fundamentals of Nursing Under the Controlled Substances Act, every dose of a controlled drug dispensed by the pharmacy must be counted for, whether the dose was administered to a particular patient or discarded accidentally. Fundamentals of Nursing A nurse can’t perform duties that violate a rule or regulation established by a state licensing board even if it is authorized by a health care facility or doctor. Fundamentals of Nursing The nurse should select a private room, preferably with a door that can be closed, to minimize interruptions during a patient interview. Fundamentals of Nursing In categorizing nursing diagnosis, the nurse should address actual life-threatening problems first, followed by potentially life-threatening concerns. Fundamentals of Nursing The major components of a nursing care plan are outcome criteria (patient goals) and nursing interventions. Fundamentals of Nursing Standing orders, or protocols, establish guidelines for treating a particular disease or set of symptoms. Fundamentals of Nursing In assessing a patient’s heart, the nurse normally finds the point of maximal impulse at the fifth intercostals space near the apex. Fundamentals of Nursing The S1 sound heard on auscultation is caused by closure of the mitral and tricuspid valves. Fundamentals of Nursing To maintain package sterility, the nurse should open the wrapper’s top flap away from the body, open side flap by touching only the outer part of the wrapper, and open the final flap by grasping the turned-down corner and pulling it toward the body. Fundamentals of Nursing The nurse shouldn’t use a cotton-tipped applicator to dry a patient’s ear canal or remove wax because it may force cerumen against the tympanic membrane. Fundamentals of Nursing A patient’s identification bracelet should remain in place until the patient has been discharged from the health care facility and has left the premises. Fundamentals of Nursing The Controlled Substances Act designated five categories, or schedules, that classify controlled drugs according to their abuse liability. Fundamentals of Nursing Schedule I drugs, such as heroin, have a high abuse potential and have no currently accepted medical use in the United States. Fundamentals of Nursing Schedule II drugs, such as morphine, opium, and meperidine (Demerol), have a high abuse potential but have currently accepted medical uses. Their use may lead to physical or psychological dependence. Fundamentals of Nursing Schedule III drugs, such as paregoric and butabarbital (Butisol), have a lower abuse potential than Schedule I or II drugs. Abuse of Schedule III drugs may lead to moderate or low physical or psychological dependence, or both. Fundamentals of Nursing Schedule IV drugs, such as chloral hydrate, have a low abuse potential compared with Schedule III drugs. Fundamentals of Nursing Schedule V drugs, such as cough syrups that contain codeine, have the lowest abuse potential of the controlled substances. Fundamentals of Nursing Activities of daily living are actions that the patient must perform every day to provide self-care and interact with society. Fundamentals of Nursing Testing of the six cardinal fields of gaze evaluates the function of all extraocular muscles and cranial nerves III, IV, and VI. Fundamentals of Nursing The six types of heart murmurs are graded from 1 to 6. A grade 6 heart murmur can be heard with stethoscope slightly raised from the chest. Fundamentals of Nursing The most important goal to include in a care plan is the patient’s goal. Fundamentals of Nursing Fruits are high in fiber and low in protein and should be omitted from a low-residue diet. Fundamentals of Nursing The nurse should use an objective scale to assess and quantify pain because postoperative pain varies greatly among individuals. Fundamentals of Nursing Postmortem care includes cleaning and preparing the deceased patient for family viewing, arranging transportation to the morgue or funeral home, and determining the disposition of belongings. Fundamentals of Nursing The nurse should provide honest answers to the patient’s questions. Fundamentals of Nursing Milk shouldn’t be included in a clear liquid diet. Fundamentals of Nursing Consistency in nursing personnel is paramount when caring for a child, and infant, or a confused patient. Fundamentals of Nursing The hypothalamus secretes vasopressin and oxytocin, which are stored in the pituitary gland. Fundamentals of Nursing The three membranes that enclose that brain and spinal cord are the dura mater, pia mater, and arachnoid. Fundamentals of Nursing A nasogastric tube is used to remove fluid and gas from the small intestine preoperatively or postoperatively. Fundamentals of Nursing Psychologists, physical therapists, and chiropractors aren’t authorized to write prescriptions for medication. Fundamentals of Nursing The area around a stoma should be cleaned with mild soap and water. Fundamentals of Nursing Vegetables have a high fiber content. Fundamentals of Nursing The nurse should use a tuberculin syringe to administer an S.C. injection of less than 1 ml. Fundamentals of Nursing For adults, S.C. injections require a 25G 1" needle; for infants, children, elderly, or very thin patients, they require a 25G to 27G ½" needle. Fundamentals of Nursing Before administering medication, the nurse should identify the patient by checking the identification band and asking the patient to state his name. Fundamentals of Nursing To clean the skin before an injection, the nurse should use a sterile alcohol swab and wipe from the center of the site outward in a circular motion. Fundamentals of Nursing The nurse always should inject heparin deep into S.C. tissue at a 90-degree angle (perpendicular to the skin) to prevent skin irritation. Fundamentals of Nursing If blood is aspirated into the syringe before an I.M. injection, the nurse should withdraw the needle, prepare another syringe, and repeat the procedure. Fundamentals of Nursing The nurse shouldn’t cut the patient’s hair without written consent from the patient or an appropriate relative. Fundamentals of Nursing If bleeding occurs after an injection, the nurse should apply pressure until the bleeding stops; if bruising occurs, the nurse should monitor the site for an enlarging hematoma. Fundamentals of Nursing When providing hair and scalp care, the nurse should begin combing at the end of the hair and work toward the head. Fundamentals of Nursing Frequency of patient hair care depends on the length and texture of the hair, duration of hospitalization, and patient’s condition. Fundamentals of Nursing Proper hearing aid function requires careful handling during insertion and removal, regular cleaning of the ear piece to prevent wax buildup, and prompt replacement of dead batteries. Fundamentals of Nursing The hearing aid marked with a blue dot is for the left ear; the one with the red dot is for the right ear. Fundamentals of Nursing A hearing aid shouldn’t be exposed to heat or humidity and shouldn’t be immersed in water. Fundamentals of Nursing The nurse should instruct a patient not to use hair spray while wearing a hearing aid. Fundamentals of Nursing The five branches of pharmacology are pharmacokinetics, pharmacodynamics, pharmacotherapeutics, toxicology, and pharmacognosy. Fundamentals of Nursing The nurse should remove heel protectors every 8 hours to inspect the foot for signs of skin breakdown. Fundamentals of Nursing The purpose of heat application is to promote vasodilation, which reduces pain caused by inflammation. Fundamentals of Nursing A sutured surgical incision is an example of healing by first intention (healing directly, without granulation). Fundamentals of Nursing Healing by secondary intention (healing by granulation) is closure of the wound by the granulation tissue filling the defect and allowing reepithelialization to occur, beginning at the wound edges and continuing to the center, until the entire wound is covered. Fundamentals of Nursing Keloid formation is an abnormality in healing characterized by overgrowth of scar tissue at the wound site. Fundamentals of Nursing The nurse should administer procaine penicillin by deep I.M. injection in the upper outer portion of the buttocks in the adult or in the midlateral thigh in the child. The nurse shouldn’t massage the injection site. Fundamentals of Nursing The ascending colostomy drains fluid feces; the descending colostomy drains solid fecal matter. Fundamentals of Nursing A folded towel (called a scrotal bridge) can provide scrotal support for the patient with scrotal edema caused by vasectomy, epididymitis, or orchitis. Fundamentals of Nursing When giving an injection to the patient with a bleeding disorder, the nurse should use a small-gauge needle and apply pressure to the site for 5 minutes after the injection. Fundamentals of Nursing Platelets are the smallest and most fragile formed element of the blood and are essential for coagulation. Fundamentals of Nursing To insert a nasogastric tube, the nurse should first instruct the patient to tilt the head back slightly and then insert the tube. When the tube is felt curving at the pharynx, the nurse should tell the patient to tilt the head forward to close the trachea and open the esophagus by swallowing. (Sips of water can facilitate this action.) Fundamentals of Nursing According to families whose loved ones are in intensive care units, their four most important needs are to have questions answered honestly, to be assured that the best possible car is being provided, to know the prognosis, and to feel there is hope. Fundamentals of Nursing A double-bind communication when the verbal message contradicts the nonverbal message and the receiver is unsure of which message to respond to. Fundamentals of Nursing A nonjudgmental attitude displayed by the nurse demonstrates that she neither approves nor disapproves of the patient. Fundamentals of Nursing Target symptoms are those that the patient and others find most distressing. Fundamentals of Nursing Advise the patient to take aspirin on an empty stomach with a full glass of water and to avoid foods with acid such as coffee, citrus fruits, and cola. Fundamentals of Nursing For every patient problem, there is a nursing diagnosis; for every nursing diagnosis, there is a goal; and for every goal, there are interventions designed to make the goal a reality. The keys to answering examination questions correctly are identifying the problem presented, formulating a goal for that specific problem, and then selecting the intervention from the choices provided that will enable the patient to reach that goal. Fundamentals of Nursing Fidelity means loyalty and can be shown as a commitment to the profession of nursing and to the patient. Fundamentals of Nursing Giving an I.M. injection against the patient’s will and without legal authority is battery. Fundamentals of Nursing An example of a third-party payor is an insurance company. Fundamentals of Nursing On-call medication should be given within 5 minutes of receipt of the call. Fundamentals of Nursing Generally, the best method to determine the cultural or spiritual needs of the patient is to ask him. Fundamentals of Nursing An incident report shouldn’t be made part of the patient’s record but is an in-house document for the purpose of correcting the problem. Fundamentals of Nursing Critical pathways are a multidisciplinary guideline for patient care. Fundamentals of Nursing When prioritizing nursing diagnoses, use this hierarchy: (1) problems associated with airway, (2) those concerning breathing, and (3) those related to circulation. Fundamentals of Nursing The two nursing diagnoses with the highest priority that the nurse can assign are Ineffective airway clearance and Ineffective breathing pattern. Fundamentals of Nursing A subjective sign that a sitz bath has been effective is that patient expresses a decrease in pain or discomfort. Fundamentals of Nursing For the nursing diagnosis Diversional activity deficit to be valid, the patient must make the statement that he’s “bored, there is nothing to do” or words to that effect. Fundamentals of Nursing The most appropriate nursing diagnosis for an individual who doesn’t speak English is Communication, impaired, related to inability to speak dominant language (English). Fundamentals of Nursing The family of the patient who has been diagnosed as hearing impaired should be instructed to face the individual when they speak to him. Fundamentals of Nursing Up to age 3, the pinna should be pulled down and back to straighten the eustachian tube before instilling medication. Fundamentals of Nursing When administering eyedrops, the nurse should waste the first drop and instill the medication in the lower conjunctival sac to prevent injury to the cornea. Fundamentals of Nursing When administering eye ointment, the nurse should waste the first bead of medication and then apply the medication from the inner to the outer canthus. Fundamentals of Nursing When removing gloves and mask, the gloves, which most likely contain pathogens and are soiled, should be removed first. Fundamentals of Nursing Crutches should placed 6" (15 cm) in front of the patient and 6" to the side to assume a tripod position. Fundamentals of Nursing Listening is the most effective communication technique. Fundamentals of Nursing Process recording is a method of evaluating one’s communication effectiveness. Fundamentals of Nursing When feeding the elderly, limit high-carbohydrate foods because of the risk of glucose intolerance. Fundamentals of Nursing Passive range of motion maintains joint mobility whereas resistive exercises increase muscle mass. Fundamentals of Nursing Isometric exercises are performed on an extremity in a cast. Fundamentals of Nursing A back rub is an example of the gate-control theory of pain. Fundamentals of Nursing Anything below the waist is considered unsterile, a sterile field becomes unsterile when it comes in contact with nay unsterile item, a sterile field must be continuously monitored, and the 1" (2.5 cm) border around a sterile field is considered unsterile. Fundamentals of Nursing A “shift to the left” is evident when there is an increase in immature cells (bands) in the blood to fight an infection. Fundamentals of Nursing A “shift to the right” is evident when there is an increase in mature cells in the blood as seen in advanced liver diseases and pernicious anemia. Fundamentals of Nursing Before administering preoperative medication, make sure that an informed consent form has been signed and attached to the patient’s record. Fundamentals of Nursing The nurse should spend no more than 30 minutes per 8-hour shift in providing care to the patient with a radiation implant. Fundamentals of Nursing The nurse should stand near the patient’s shoulders for cervical implants and at the foot of the bed for head and neck implants. Fundamentals of Nursing The nurse should never be assigned to care for more than one patient with radiation implants. Fundamentals of Nursing Long-handled forceps and a lead-lined container should be in the room of the patient who has a radiation implant. Fundamentals of Nursing Generally, patients who have the same infection and are in strict isolation can share the same room. Fundamentals of Nursing Diseases requiring strict isolation include chickenpox, diphtheria, and viral hemorrhagic fever such as Marburg virus disease. Fundamentals of Nursing For the patient abiding by Jewish custom, milk and meat shouldn’t be served in the same meal. Fundamentals of Nursing Whether the patient can perform a procedure (psychomotor domain of learning) is a better indicator of the effectiveness of patient teaching than whether the patient can simply state the steps of the procedure (cognitive domain of learning). Fundamentals of Nursing Developmental stages according to Erik Erikson are trust versus mistrust (birth to 18 months), autonomy versus shame and doubt (18 months to 3 years), initiative versus guilt (3 to 5 years), industry versus inferiority (5 to 12 years), identity versus identity diffusion (12 to 18 years), intimacy versus isolation (18 to 25 years), generativity versus stagnation (25 to 60 years), and ego integrity versus despair (older than 60 years). Fundamentals of Nursing Face the hearing impaired patient when communicating with him. Fundamentals of Nursing A proper nursing intervention for the spouse of the patient who has suffered a serious incapacitating disease is to assist him in mobilizing a support system. Fundamentals of Nursing Hyperpyrexia refers to extreme elevation in temperature above 106º F (41.1º C). Fundamentals of Nursing

Basic Drills

1.According to Maslow, which of the following categories of needs represents the most basic?

a. Physiologic needs
b.Self-actualization
c. Safety and security needs
d. Belongingness

2.Which of the following statements reflects the World Health Organization’s definition of health?

a. A state of complete physical, mental, and social well-being and not merely the absence of disease and infirmity.
b. A condition of homeostatis and adaptation.
c. An individual’s location along a wellness--illness continuum.
d. A fluid, ever-changing balance reflected through physical, mental, and social behavior.

3.Which of the following statements defines culture?

a. The learned patterns of behavior, beliefs, and values that can be attributed to a particular group of people.
b. A group of people distinguished by genetically transmitted material.
c. The status of belonging to a particular region by origin, birth, or naturalization.
d. The classification of a group based upon certain distinctive characteristics.

4.The reason that case management has gained such prominence in health care can be traced to

a. decreased cost of care associated with inpatient stay.
with rapid and frequent inter-unit transfers from specialty to standard care units.

b. increased length of hospital stay.
c. discharge from specialty care units to home.
d. limited availability for inter-unit hospital transfers.

5.A preferred provider organization is described as a

a. business arrangement between hospitals and physicians.
b. prepaid group health practice system.
c. limited insurance program.
d. health care savings account program.

6.Which of the following categories identifies the focus of community/public health nursing practice?

a. Promoting and maintaining the health of populations and preventing and minimizing the progress of disease
b. Rehabilitation and restorative services
c. Adaptation of hospital care to the home environment
d. Hospice care delivery

7.A major goal for home care nurses is

a. restoring maximum health function.
b. promoting the health of populations.
c. minimizing the progress of disease.
d. maintaining the health of populations.

8.In the United States, nurses performing invasive procedures need to be up-to-date with their immunizations, particularly

a. hepatitis B.
b. hepatitis E..
c. hepatitis A.
d. hepatitis C.

9.At what time during a patient’s hospital stay does discharge planning begin?

a. Admission
b. Twenty-four hours prior to discharge
c. The shift prior to discharge
d. By the third hospital day

10.The leading health problems of elementary school children include

a. cancer.-
b. alcohol and drug abuse.
c. mental and emotional problems
d.homicide.

11.Which skill needed by the nurse to think critically involves identification of patient problems indicated by data?

a. Analysis
b. Interpretation
c. Inferencing
d. Explanation

12.The ethics theory that focuses on ends or consequences of actions is the

a. utilitarian theory.
b. formalist theory.
c. deontological theory.
d. adaptation theory.

13.Which of the following ethical principles refers to the duty to do good?

a. Beneficence
b. Fidelity
c. Veracity
d. Nonmaleficence

14.During which step of the nursing process does the nurse analyze data related to the patient's health status?

a. Assessment
b. Implementation
c. Diagnosis
d. Evaluation

15.The basic difference between nursing diagnoses and collaborative problems is that

a. nurses manage collaborative problems using physician-prescribed interventions.
b. collaborative problems can be managed by independent nursing interventions.
c. nursing diagnoses incorporate physician-prescribed interventions.
d. nursing diagnoses incorporate physiologic complications that nurses monitor to detect change in status.

16.Health education of the patient by the nurseis an independent function of nursing practice.

a. Health education is an independent function of nursing practice and is included in all state nurse practice acts.
b. requires a physician's order.
c. must be approved by the physician.
d. must focus on wellness issues.

17.Nonadherence to therapeutic regimens is a significant problem for which of the following age groups?

a. Adults 65 and over
b. Teenagers
c. Children
d. Middle-aged adults

18.Experiential readiness to learn refers to the patient'spast history with education and life experience.

a. Experiential readiness refers to past experiences that influence a person's ability to learn.
b. emotional status.
c. acceptance of an existing illness.
d. ability to focus attention.

19.Asking the patient questions to determine if the person understands the health teaching provided would be included during which step of the nursing process?

a. Evaluation
b. Assessment
c. Planning and goals
d. Implementation

20.Which of the following items is considered the single most important factor in assisting the health professional in arriving at a diagnosis or determining the person's needs?

a. History of present illness
b. Physical examination
c. Diagnostic test results
d. Biographical data

21.Of the following areas for assessing the patient profile, which should be addressed after the others?

a. Body image
b. Education
c. Occupation
d. Environment

22.Which of the following methods of physical examination refers to the translation of physical force into sound?

a. Percussion
b. Palpation
c. Auscultation
d. Manipulation

23.In which range of body mass index (BMI) are patients considered to have increased risk for problems associated with poor nutritional status?

a. Below 24
b. 25-29
c. 30 to 34
d. Over 40

24.To calculate the ideal body weight for a woman, the nurse allows

a. 100 pounds for 5 feet of height.
b. 106 pounds for 5 feet of height.
c. 6 pounds for each additional inch over 5 feet.
d. 80 pounds for 5 feet of height.

25. A steady state within the body is termed
a. homeostasis.
b. constancy.
c. adaptation.
d. stress.

26.Which of the following terms, according to Lazarus, refers to the process through which an event is evaluated with respect to what is at stake and what might and can be done?

a. Cognitive appraisal
b. Coping
c. Hardiness
d. Adaptation

27.An increase in the number of new cells in an organ or tissue that is reversible when the stimulus for production of new cells is removed is termed

a. hyperplasia.
b. hypertrophy.
c. atrophy.
d. neoplasia.

28. Which of the following types of cells have a latent ability to regenerate?

a. Stable
b. Labile
c. Permanent
d. Epithelial

29.The relaxation techniques of progressive muscle relaxation, relaxation with guided imagery, and the Benson Relaxation Response share which of the following elements?

a. A mental device (something on which to focus the attention)
b. Nutritional foundation
c. Analgesic preparation
d. Physician's order

30.Which of the following terms has been defined by the American Psychiatric Association as a group of behavioral or psychological symptoms or a pattern that manifests itself in significant distress, impaired functioning, or accentuated risk of enduring severe suffering or possible death?

a. Mental disorder
b. Emotional disorder
c. Anxiety
d. Schizophrenia

31.Establishing financial security has been identified as a developmental task of which of the following groups?

a. Middle adult
b. Older adult
c. Young adult
d. Teenager

32.When up to a 6-month period elapses between the experience of trauma and the onset of symptoms of posttraumatic stress disorder (PTSD), the episode is termed

a. delayed.
b. acute.
c. chronic.
d. primary.


33.Which of the following statements accurately describes a risk factor for depression?

a. History of physical or sexual abuse
b. Male gender
c. Age over 50 years
d. Negative family history of depression

34.Of the following stages of grieving as described by Kubler-Ross, which is the initial?

a. Denial
b. Anger
c. Bargaining
d. Depression

35.Which of the following terms refers to Leininger's description of the learned and transmitted knowledge about values, beliefs, rules of behavior, and lifestyle practices that guide a designated group in their thinking and actions in patterned ways?

a. Culture
b. Minority
c. Race
d. Subculture

36.The inability of a person to recognize his or her own values, beliefs, and practices and those of others because of strong ethnocentric tendencies is termed.

a. cultural blindness.
b. acculturation.
c. cultural imposition.
d. cultural taboo.

37. Which of the following groups of individuals may stare at the floor during conversations as a sign of respect?

a. Native Americans
b. Indo-Chinese
c. Arabs
d. Asians



38.For which of the following religious groups is all meat prohibited?

a. Hinduism
b. Seventh-Day Adventism
c. Judaism
d. Islam

39.The paradigm that explains the cause of illness as an imbalance in the forces of nature is the

a. holistic perspective.
b. magico-religious view.
c. biomedical view.
d. scientific view.

40.The aim of genomic medicine is

a. improving predictions about individuals’ susceptibility to diseases
b. reproduction
c. cure of disease
d. cloning

41.Nondisjunction of a chromosome results in which of the following diagnoses?

a. Down Syndrome
b. Huntingon Disease
c. Duchenne Muscular Dystrophy
d. Marphan Syndrome

42.Which type of Mendelian inherited condition results in both genders being affected equally in a vertical pattern?

a. Automosomal dominant inheritance
b. Automosomal recessive inheritance
c. X-linked inheritance.
d. Multifactorial genetic inheritance

43.A specific BRCA1 cancer-predisposing gene mutation seems to occur more frequently among women of which descent?

a. Ashkanazi Jewish
b. Mediterranean
c. African American
d. Chinese and Japanese

44.Which of the following statements describes accurate information related to chronic illness?

a. Most people with chronic conditions do not consider themselves sick or ill.
b. Most people with chronic conditions take on a sick role identity.
c. Chronic conditions do not result from injury.
d. Most chronic conditions are easily controlled.

45.In which phase of the trajectory model of chronic illness are the symptoms under control and managed?

a. Stable
b. Acute
c. Comeback
d. Downward

46.Which phase of the trajectory model of chronic illness is characterized by reactivation of the illness?

a. Unstable
b. Stable
c. Acute
dComeback
47.Which phase of the trajectory model of chronic illness is characterized by the gradual or rapid decline in the trajectory despite efforts to halt the disorder?

a. Dying
b. Unstable
c. Acute
d. Downward

48.In order to help prevent the development of an external rotation deformity of the hip in a patient who must remain in bed for any period of time, the most appropriate nursing action would be to use

a. a trochanter roll extending from the crest of the ilium to the midthigh.
b. pillows under the lower legs.
c. a hip-abductor pillow.
d. a footboard.



49.To prevent footdrop, the patient is positioned in:

a. Order to keep the feet at right angles to the leg
b. A semi-sitting position in bed
c. A sitting position with legs hanging off the side of the bed
d. A side-lying position

50.Through which of the following activities does the patient learn to consciously contract excretory sphincters and control voiding cues?

a. Biofeedback
b. Kegel exercises
c. Habit training
d. Bladder training

51.During which stage of pressure ulcer development does the ulcer extend into the subcutaneous tissue?

a. Stage III
b. Stage IV
c. Stage II
d. Stage I

52.During which stage of pressure ulcer development does the ulcer extend into the underlying structures, including the muscle and possibly the bone?

a. Stage IV
b. Stage III
c. Stage II
d. Stage I

53.Which type of incontinence is associated with weakened perineal muscles that permit leakage of urine when intra-abdominal pressure is increased?

a. Stress incontinence
b. Urge incontinence
c. Reflex (neurogenic) incontinence
d. Functional incontinence

54.Ageism refers to

a. Bias against older people based solely on chronological age
b. fear of old age.
c. loss of memory.
d. benign senescent forgetfulness.

55.When assessing the older adult, the nurse anticipates increase in which of the follow components of respiratory status?

a. Residual lung volume
b. Vital capacity
c. Gas exchange and diffusing capacity
d. Cough efficiency

56.According to the classification of hypertension diagnosed in the older adult, hypertension that can be attributed to an underlying cause is termed

a. secondary.
b. primary.
c. essential.
d. isolated systolic.

57.Which of the following terms refers to the decrease in lens flexibility that occurs with age, resulting in the near point of focus getting farther away?

a. Presbyopia
b. Presbycusis
c. Cataract
d. Glaucoma

58.Which of the following states is characterized by a decline in intellectual functioning?

a. Dementia
b.Depression
c. Delirium
d. Delusion

59.When a person who has been taking opioids becomes less sensitive to their analgesic properties, that person is said to have developed a (an)

a. tolerance.
b. addiction.
c. dependence.
d. balanced analgesia.



60.Prostaglandins are chemical substances thought to

a. increase sensitivity of pain receptors.
b. reduce the perception of pain.
c. inhibit the transmission of pain.
d. inhibit the transmission of noxious stimuli.

61.Which of the following principles or guidelines accurately informs the nurse regarding placebos?

a. Placebos should never be used to test the person's truthfulness about pain.
b. A placebo effect is an indication that the person does not have pain.
c. A placebo should be used as the first line of treatment for the patient.
d. A positive response to a placebo indicates that the person's pain is not real.

62.Regarding tolerance and addiction, the nurse understands that

a. although patients may need increasing levels of opioids, they are not addicted.
b. tolerance to opioids is uncommon.
c. addiction to opioids commonly develops.
d. the nurse must be primarily concerned about development of addiction by the patient in pain.

63.The preferred route of administration of medication in the most acute care situations is which of the following routes?

a. Intravenous
b. Epidural
c. Subcutaneous
d. Intramuscular

64.Mu opioids have which of the following effects on respiratory rate:

a. Stimulation, then depression
b. No change
c. Stimulation, only
d. Depression, only

65.Which of the following electrolytes is a major cation in body fluid?

a. Potassium
b. Chloride
c. Bicarbonate
d. Phosphate

66.Which of the following electrolytes is a major anion in body fluid?

a. Chloride
b. Potassium.
c.Sodium
c. Calcium

67.Oncotic pressure refers to

a. the osmotic pressure exerted by proteins.
b. the number of dissolved particles contained in a unit of fluid.
c. the excretion of substances such as glucose through increased urine output.
d.the amount of pressure needed to stop flow of water by osmosis.

68.Which of the following solutions is hypotonic?

a. 0.45% NaCl.
b. Lactated Ringer's solution.
c. 0.9% NaCl.
d. 5% NaCl.

69.The normal serum value for potassium is

a. 3.5-5.5 mEq/L.
b. 135-145 mEq/L.
c. 96-106 mEq/L.
d. 8.5-10.5 mg/dL.

70.In which type of shock does the patient experiences a mismatch of blood flow to
the cells?

a. Distributive
b. Cardiogenic
c. Hypovolemic
d. Septic

71.Which stage of shock is best described as that stage when the mechanisms that regulate blood pressure fail to sustain a systolic pressure above 90 mm Hg?

a. Progressive
b. Refractory
c. Compensatory
d. Irreversible

72.When the nurse observes that the patient's systolic blood pressure is less than 80--90 mm Hg, respirations are rapid and shallow, heart rate is over 150 beats per minute, and urine output is less than 30 cc per hour, the nurse recognizes that the patient is demonstrating which stage of shock?

a. Compensatory
b. Progressive
c. Refractory
d. Irreversible

73.Which of the following vasoactive drugs used in treating shock results in reduced preload and afterload, reducing oxygen demand of the heart?

a. Nitroprusside (Nipride)
b. Dopamine (Intropin)
c. Epinephrine (adrenaline)
d. Methoxamine (Vasoxyl)

74.The nurse anticipates that the immunosuppressed patient is at greatest risk for which type of shock?

a. Septic
b. Neurogenic
c. Cardiogenic
d. Anaphylactic

75.Which of the following colloids is expensive but rapidly expands plasma volume?

a. Albumin
b. Dextran
c. Lactated Ringers
d. Hypertonic Saline

76.Which of the following terms refers to cells that lack normal cellular characteristics and differ in shape and organization with respect to their cells of origin?

a. Anaplasia
b. Neoplasia
c. Dysplasia
d. Hyperplasia

77.Palliation refers to

a. relief of symptoms associated with cancer.
b. hair loss.
c. the spread of cancer cells from the primary tumor to distant sites.
d. the lowest point of white blood cell depression after therapy that has toxic effects on the bone marrow.

78.During which step of cellular carcinogenesis do cellular changes exhibit increased malignant behavior?

a. Progression
b. Promotion
c. Initiation
d. Prolongation

79.The drug, Interleukin-2, is an example of which type of biologic response modifier?

a. Cytokine
b. Monoclonal antibodies
c. Retinoids
d. Antimetabolites

80.Of the following terms, which is used to refer to the period of time during which mourning a loss takes place?

a. Bereavement
b. Grief
c. Mourning
c. Hospice

81.Which of the following "awareness contexts" is characterized by the patient, the family, and the health care professionals being aware that the patient is dying but all pretend otherwise?

a. Mutual pretense awareness
b. Closed awareness
c. Suspected awareness
d. Open awareness

82.For individuals known to be dying by virtue of age and/or diagnoses, which of the following signs indicate approaching death:

a. Increased restlessness
b.Increased wakefulness
c. Increased eating
d. Increased urinary output

83.Which of the following terms best describes a living will?

a. Medical directive
b. Proxy directive
c. Health care power of attorney
d. Durable power of attorney for health


84.A malignant tumor

a. gains access to the blood and lymphatic channels.
b. demonstrates cells that are well-differentiated.
c. is usually slow growing.
d. grows by expansion.

85.Which of the following classes of antineoplastic agents is cell--cycle-specific?

a. Antimetabolites (5-FU)
b. Antitumor antibiotics (bleomycin)
c. Alkylating agents (cisplatin)
b. Nitrosureas (carmustine)

86.Regarding the surgical patient, which of the following terms refers to the period of time that constitutes the surgical experience?

a. Perioperative phase
b. Preoperative phase
c. Intraoperative phase
d. Postoperative phase

87.When the indication for surgery is without delay, the nurse recognizes that the surgery will be classified as

a. emergency.
b. urgent.
c. required.
d. elective.

88.When a person with a history of chronic alcoholism is admitted to the hospital for surgery, the nurse anticipates that the patient may show signs of alcohol withdrawal delirium during which time period?

a. Up to 72 hours after alcohol withdrawal
b. Immediately upon admission
c. Upon awakening in the post-anesthesia care unit
d. Up to 24 hours after alcohol withdrawal

89.Which of the following categories of medications may result in seizure activity if withdrawn suddenly?

a.Tranquilizers
b. Adrenal corticosteroids
c. Antidepressants
d. Diuretics

90.When the patient is encouraged to concentrate on a pleasant experience or restful scene, the cognitive coping strategy being employed by the nurse is

a. imagery.
b. optimistic self-recitation.
c. distraction.
d. progressive muscular relaxation.

91.According to the American Society of Anesthesiology Physical Status Classification System, a patient with severe systemic disease that is not incapacitating is noted to have physical status classification

a. P3
b. P4
c. P1
d. P2

92.Which stage of anesthesia is termed surgical anesthesia?

a. III
b. I
c. II
d. IV

93.Fentanyl (Sublimaze) is categorized as which type of intravenous anesthetic agent?

a. Neuroleptanalgesic
b. Tranquilizer
c. Opioid
d. Dissociative agent

94Which of the following manifestations is often the earliest sign of malignant hyperthermia?

a. Tachycardia (heart rate above 150 beats per minute)
b. Hypotension
c. Elevated temperature
d. Oliguria

95.Which of the following terms is used to refer to protrusion of abdominal organs through the surgical incision?

a. Evisceration
b. Hernia
c. Dehiscence
d. Erythema

96.When the method of wound healing is one in which wound edges are not surgically approximated and integumentary continuity is restored by granulations, the wound healing is termed

a. second intention healing.
b. primary intention healing.
c. first intention healing.
d. third intention healing.

97.The nurse recognizes which of the following signs as typical of the patient in shock?

a. Rapid, weak, thready pulse
b. Flushed face
c. Warm, dry skin
d. Increased urine output

98.When the nurse observes that the postoperative patient demonstrates a constant low level of oxygen saturation, although the patient's breathing appears normal, the nurse identifies that the patient may be suffering which type of hypoxemia?

a. Subacute
b. Hypoxic
c. Episodic
d. Anemic

99.When the surgeon performs an appendectomy, the nurse recognizes that the surgical category will be identified as

a. clean contaminated.
b. clean.
c. contaminated.
d. dirty.

100.Which of the following terms is used to describe inability to breathe easily except in an upright position?

a. Orthopnea
b. Dyspnea
c. Hemoptysis
d. Hypoxemia

101.In relation to the structure of the larynx, the cricoid cartilage is

a. the only complete cartilaginous ring in the larynx.
b. used in vocal cord movement with the thyroid cartilage.
c. the largest of the cartilage structures.
d. the valve flap of cartilage that covers the opening to the larynx during swallowing.

102.Which respiratory volume is the maximum volume of air that can be inhaled after maximal expiration?

a. Inspiratory reserve volume
b. Tidal volume
c. Expiratory reserve volume
d. Residual volume

103.The individual who demonstrates displacement of the sternum is described as having a

a. pigeon chest.
b. barrel chest.
c. funnel chest.
d. kyphoscoliosis.

104.When the nurse auscultates chest sounds that are harsh and cracking, sounding like two pieces of leather being rubbed together, she records her finding as

a. pleural friction rub.
b. crackles.
c. sonorous wheezes.
d. sibilant wheezes.

105.Which of the following terms is used to describe hemorrhage from the nose?

a. Epistaxis
b. Xerostomia
c.Rhinorrhea
d. Dysphagia

106.The herpes simplex virus (HSV-1), which produces a cold sore (fever blister), has an incubation period of

a. 2-12 days.
b. 0-3 months.
c. 20-30 days.
d. 3-6 months.

107.Another term for clergyman's sore throat is

a. chronic granular pharyngitis.
b. aphonia.
c. atrophic pharyngitis.
d. hypertrophic pharyngitis.

108.Which type of sleep apnea is characterized by lack of airflow due to pharyngeal occlusion?

a. Obstructive
b. Simple
c. Mixed
d. Central

109.When the patient who has undergone laryngectomy suffers wound breakdown, the nurse monitors him very carefully because he is identified as being at high risk for

a. carotid artery hemorrhage.
b. pulmonary embolism.
c. dehydration.
d. pneumonia.

110.Which of the following terms refers to lung tissue that has become more solid in nature due to a collapse of alveoli or infectious process?

a. Consolidation
b. Atelectasis
c. Bronchiectasis
d. Empyema

111.Which of the following community-acquired pneumonias demonstrates the highest occurrence during summer and fall?

a. Legionnaires' disease
b. Streptococcal (pneumococcal) pneumonia
c. Mycoplasma pneumonia
d. Viral pneumonia

112.When interpreting the results of a Mantoux test, the nurse explains to the patient that a reaction occurs when the intradermal injection site shows

a. redness and induration.
b. drainage.
c. tissue sloughing.
d. bruising.

113.Which of the following actions is most appropriate for the nurse to take when the patient demonstrates subcutaneous emphysema along the suture line or chest dressing 2 hours after chest surgery?

a. Record the observation..
b. Apply a compression dressing to the area.
c. Measure the patient's pulse oximetry.
d. Report the finding to the physician immediately.

114.Which of the following types of lung cancer is characterized as fast growing and tending to arise peripherally?

a. Large cell carcinoma
b. Bronchioalveolar carcinoma
c. Adenocarcinoma
d. Squamous cell carcinoma

115.Which of the following methods is the best method for determining nasogastric tube placement in the stomach?

a. X-ray
b. Observation of gastric aspirate
c. Testing of pH of gastric aspirate
d. Placement of external end of tube under water

116.Which of the following types of lung cancer is the most prevalent carcinoma of the lung for both men and women?

a. Adenocarcinoma
b. Large cell carcinoma
c. Squamous cell carcinoma
d. Small cell carcinoma

117.Emphysema is described as:

a. A disease of the airways characterized by destruction of the walls of overdistended alveoli.
b. A disease that results in a common clinical outcome of reversible airflow obstruction.
c. The presence of cough and sputum production for at least a combined total of two or three months in each of two consecutive years.
d. Chronic dilatation of a bronchus or bronchi

118.Which of the following is the most important risk factor for development of Chronic Obstructive Pulmonary Disease?

a. Cigarette smoking
b. Occupational exposure
c. Air pollution
d. Genetic abnormalities

119.Which type of chest configuration is typical of the patient with COPD?

a. Barrel chest
b. Pigeon chest
c. Flail chest
d. Funnel chest


120.In which stage of COPD is the forced expiratory volume (FEV1) < 30%?

a. III
b. II
c. I
d. O

121.Of the following oxygen administration devices, which has the advantage of providing high oxygen concentration?

a. Non-rebreather mask
b. Venturi mask
c. Catheter
d. Face tent

122.Which of the following ranges identifies the amount of pressure within the endotracheal tube cuff that is believed to prevent both injury and aspiration?

a. 20-25 mm Hg water pressure.
b. 10-15 mm Hg water pressure.
c. 30-35 mm Hg water pressure.
d. 0-5 mm Hg water pressure

123.When performing endotracheal suctioning, the nurse applies suctioning while withdrawing and gently rotating the catheter 360 degrees for which of the following time periods?

a. 10-15 seconds
b. 30-35 seconds
Applying suction for 30-35 seconds is hazardous and may result in the patient's developing hypoxia, which can lead to dysrhythmias and, ultimately, cardiac arrest.
c. 20-25 seconds
d. 0-5 seconds

124.In general, chest drainage tubes are not used for the patient undergoing

a. Pneumonectomy
b. Lobectomy
c. Wedge resection
d. Segmentectomy

125.Which term is used to describe the ability of the heart to initiate an electrical impulse?

a. Automaticity
b. Contractility
c. Conductivity
d. Excitability

126.The nurse auscultates the apex beat at which of the following anatomical locations?

a. Fifth intercostal space, midclavicular line
b. Mid-sternum
c. 2” to the left of the lower end of the sternum
d. 1” to the left of the xiphoid process

127.Which of the following terms describes the amount of blood ejected per heartbeat?

a. Stroke volume
b. Cardiac output
c. Ejection fraction
d. Afterload

128.When measuring the blood pressure in each of the patient's arms, the nurse recognizes that in the normal adult, the pressures

a. differ no more than 5 mm Hg between arm pressures.
b. must be equal in both arms.
c. may vary 10 mm Hg or more between arms.
d. may vary, with the higher pressure found in the left arm.

129.Central venous pressure is measured in which of the following heart chambers?

a. Right atrium
b. Left atrium
c. Left ventricle
d. Right ventricle

130.Which of the following ECG characteristics is usually seen when a patient's serum potassium level is low?

a. U wave
b. T wave
c. P wave
d. QT interval

131.Which of the following ECG waveforms characterizes conduction of an electrical impulse through the left ventricle?

a. QRS complex
b. P wave
c. PR interval
d. QT interval

132.When the nurse observes that the patient's heart rate increases during inspiration and decreases during expiration, the nurse reports that the patient is demonstrating

a. sinus dysrhythmia.
b. normal sinus rhythm.
c. sinus bradycardia.
d. sinus tachycardia.

133.Which of the following terms is used to describe a tachycardia characterized by abrupt onset, abrupt cessation, and a QRS of normal duration?

a. Paroxysmal atrial tachycardia
b. Sinus tachycardia
c. Atrial flutter
d. Atrial fibrillation

134.When the nurse observes an ECG tracing on a cardiac monitor with a pattern in lead II and observes a bizarre, abnormal shape to the QRS complex, the nurse has likely observed which of the following ventricular dysrhythmias?

a. Premature ventricular contraction
b. Ventricular bigeminy
c. Ventricular tachycardia
d. Ventricular fibrillation

135.Premature ventricular contractions are considered precursors of ventricular tachycardia when they

a. occur at a rate of more than six per minute.
b. occur during the QRS complex.
c. have the same shape.
d. are paired with a normal beat.

136.When no atrial impulse is conducted through the AV node into the ventricles, the patient is said to be experiencing which type of AV block?

a. Third degree
b. First degree
c. Second degree, type I
d. Second degree, type II

137.Which of the following terms refers to chest pain brought on by physical or emotional stress and relieved by rest or medication?

a. angina pectoris
b. atherosclerosis
c. atheroma
d. ischemia


138.Of the following risk factors, which is considered modifiable?

a. Diabetes mellitus
b. Gender
c. Race
d. Increasing age

139.When the patient with known angina pectoris complains that he is experiencing chest pain more frequently even at rest, the period of pain is longer, and it takes less stress for the pain to occur, the nurse recognizes that the patient is describing

a. unstable angina.
b. intractable angina.
c. variant angina.
d. refractory angina.

140.Heparin therapy is usually considered therapeutic when the patient's activated partial thromboplasin time (aPTT) is how many times normal?

a. 1.5 to 2
b. .5 to 1
c. 2.5 to 3
d .25 to .75

141.When the post-cardiac surgery patient demonstrates restlessness, nausea, weakness, and peaked T waves, the nurse reviews the patient's serum electrolytes anticipating which abnormality?

a. Hyperkalemia
b. Hypercalcemia
c. Hypomagnesemia
d. Hyponatremia


142.In order to be effective, Percutaneous Transluminal Coronary Angioplasty (PTCA) must be performed within what time frame, beginning with arrival at the emergency department after diagnosis of myocardial infarction?

a. 60 minutes
b. 30 minutes
c. 9 days
d. 6-12 months

143.Which of the following statements reflect a goal of rehabilitation for the patient with an MI:

a. To improve the quality of life
b. To limit the effects and progression of atherosclerosis
c. To return the patient to work and a pre illness lifestyle
d. To prevent another cardiac event

144.Which of the following methods to induce hemostasis after sheath removal post Percutaneous Transluminal Coronary Angioplasty is the least effective?

a. Application of a sandbag to the area
b. Application of a vascular closure device, such as AngiosealTM, VasosealTM, c. Direct manual pressure
d. Application of a pneumatic compression device (e.g., Fem-StopTM)

145.A long-term effect of which of the following procedures post acute MI induces angioneogenesis?

a. Transmyocardial laser revascularization
b. Bracytherapy
c. Atherectomy
d. Stent placement


146.Which of the following medications are used to reverse the effects of heparin?

a. Protamine sulfate
b. Streptokinase
c. Clopidigrel (Plavix)
d.Aspirin
147.Which of the following terms refers to leg pain that is brought on walking and caused by arterial insufficiency?

a. Intermittent claudication
b. Dyspnea
c. Orthopnea
d. Thromboangitis obliterans

148.When the post-cardiac surgical patient demonstrates vasodilation, hypotension, hyporeflexia, slow gastrointestinal motility (hypoactive bowel sounds), lethargy, and respiratory depression, the nurse suspects which of the following electrolyte imbalances?

a. Hypermagnesemia
b. Hypokalemia
c. Hyperkalemia
d. Hypomagnesemia

149.When the nurse notes that the post cardiac surgery patient demonstrates low urine output (< 25 ml/hr) with high specific gravity (> 1.025), the nurse suspects:

a. Inadequate fluid volume
b. Normal glomerular filtration
c. Overhydration
d. Anuria


150.When the valve used in valve replacement surgery is made from the patient's own heart valve, which of the following terms is used?

a. Autograft
b. Allograft
c. Homograft
d. Xenograft

151.Which of the following procedures most specifically describes splitting or separating fused cardiac valve leaflets?

a. Commisurotomy
b. Annuloplasty
c. Chordoplasty
d. Valvuloplasty

152.Which of the following mitral valve conditions generally produces no symptoms?

a. Prolapse
b.Stenosis
c. Regurgitation
d. Infection

153.In which type of cardiomyopathy does the heart muscle actually increase in size and mass weight, especially along the septum?

a. Hypertrophic
b. Dilated
c. Restrictive
d. Arrhythmogenic right ventricular cardiomyopathy



154.Which of the following patient behaviors, if observed by the nurse, would indicate that the cardiac patient's level of anxiety has decreased?

a. Answers questions regarding status with no problem.
b. Discusses prognosis freely.
c. Verbalizes fears and concerns.
d. Participates in support groups.

155.The patient with which of the follow characteristics is considered high risk for the development of infective endocarditis?

a. The patient who has complex cyanotic congenital malformations is at high risk for the development of infective endocarditis.
b. Mitral valve prolapse with valvular regurgitation
c. Hypertrophic cardiomyopathy
d. Acquired valvular dysfunction

156.Which of the following terms indicates the amount of blood pumped out of the ventricle with each contraction of the heart?

a. Stroke volume
b. Afterload
c. Cardiac output
d. Preload

157.When the balloon on the distal tip of a pulmonary artery catheter is inflated and a pressure is measured, the measurement obtained is referred to as the

a. pulmonary artery wedge pressure.
b. central venous pressure.
c. pulmonary artery pressure.
d. cardiac output.

158.Which of the following medications is categorized as a loop diuretic?

a. Furosemide (Lasix)
b. Chlorothiazide (Diuril)
c. Chlorthalidone (Hygroton)
d. Spironolactone (Aldactone)

159.When the nurse observes that the patient always has difficulty breathing when lying flat, the nurse records that the patient is demonstrating

a. Orthopnea
b. Dyspnea on exertion.
c. Hyperpnea.
d. Paroxysmal nocturnal dyspnea.

160.The patient with cardiac failure is taught to report which of the following symptoms to the physician or clinic immediately?

a. Persistent cough
b. Increased appetite
c. Weight loss
d. Ability to sleep through the night

161.A classic sign of cardiogenic shock is

a. Tissue hypoperfusion
b. High blood pressure
c. Hyperactive bowel sounds
d. Increased urinary output

162.Vasoactive drugs which cause the arteries and veins to dilate, thereby shunting much of the intravascular volume to the periphery and causing a reduction in preload and afterload include agents such as

a. Sodium nitroprusside (Nipride)
b. Norepinephrine (Levophed)
c. Dopamine (Inotropin)
d. Furosemide (Lasix)

163.Which of the following terms refers to a muscular, cramp-like pain in the extremities consistently reproduced with the same degree of exercise and relieved by rest?

a. Intermittent claudication
b. Aneurysm
c. Bruit
d. Ischemia

164.Which of the following observations regarding ulcer formation on the patient's lower extremity indicate to the nurse that the ulcer is a result of venous insufficiency?

a. The border of the ulcer is irregular.
b. The ulcer is very painful to the patient, even though superficial.
c. The ulcer base is pale to black.
d. The ulcer is deep, involving the joint space.

165.A diagnostic test that involves injection of a contrast media into the venous system through a dorsal vein in the foot is termed

a. contrast phlebography.
b. air plethysmography
c. lymphangiography.
d. lymphoscintigraphy.

166.The nurse teaches the patient with peripheral vascular disease to refrain from smoking because nicotine causes

a. vasospasm.
b. slowed heart rate.
c. depression of the cough reflex.
d. diuresis.

167.Which of the following types of aneurysms results in bleeding into the layers of the arterial wall?

a. Dissecting
b. Saccular
c. False
d. Anastomotic

168.Which of the following terms refers to enlarged, red, and tender lymph nodes?

a. Lymphadenitis
b. Lymphangitis
c. Lymphedema
d. Elephantiasis

169.Which of the following terms is given to hypertension in which the blood pressure, which is controlled with therapy, becomes uncontrolled (abnormally high) with the discontinuation of therapy?

a. Rebound
b. Essential
c. Primary
d. Secondary

170.Officially, hypertension is diagnosed when the patient demonstrates a systolic blood pressure greater than ______ mm Hg and a diastolic blood pressure greater than _____ mm Hg over a sustained period.

a. 140, 90
b. 130, 80
c. 110, 60
d. 120, 70

171.The nurse teaches the patient which of the following guidelines regarding lifestyle modifications for hypertension?

a. Maintain adequate dietary intake of potassium
b. Reduce smoking to no more than four cigarettes per day
c. Limit aerobic physical activity to 15 minutes, three times per week
d. Stop alcohol intake

172.Of the following diuretic medications, which conserves potassium?

a. Spironolactone (Aldactone)
b. Furosemide (Lasix)
c. Chlorothiazide (Diuril)
d. Chlorthalidone (Hygroton)

173.Which of the following adrenergic inhibitors acts directly on the blood vessels, producing vasodilation?

a. Prazosin hydrochloride (Minipress)
b. Reserpine (Serpasil)
c. Propranolol (Inderal)
d. Clonidine hydrochloride (Catapres)

174.Which of the following terms refers to an abnormal decrease in white blood cells, red blood cells, and platelets?

a. Pancytopenia
b. Anemia
c. Leukopenia
d. Thrombocytopenia

175.Which of the following terms refers to a form of white blood cell involved in immune response?

a. Lymphocyte
b. Granulocyte
c. Spherocyte
d. Thrombocyte

176.The term that is used to refer to a primitive cell, capable of self-replication and differentiation, is

a. stem cell.
b. band cell.
c. spherocyte.
d. reticulocyte.

177.Of the following hemolytic anemias, which is categorized as inherited?

a. Sickle cell anemia
b. Autoimmune hemolytic anemia
c. Cold agglutinin disease
d. Hypersplenism

178.The antidote to heparin is
a. protamine sulfate.
b. vitamin K.
c. Narcan.
d. Ipecac.

179.Which of the following terms describes a gastric secretion that combines with vitamin B-12 so that it can be absorbed?

a. Intrinsic factor
b. Amylase
c. Pepsin
d. Trypsin
180. When bowel sounds are heard about every 15 seconds, the nurse would record that the bowel sounds are

a. normal.
b. hypoactive.
c. sluggish.
d. absent.

181.When gastric analysis testing reveals excess secretion of gastric acid, which of the following diagnoses is supported?

a. Duodenal ulcer
b. Chronic atrophic gastritis
c. Gastric cancer
d. Pernicious anemia

182.Which of the following terms is used to describe stone formation in a salivary gland, usually the submandibular gland?

a. Sialolithiasis
b. Parotitis
c. Sialadenitis
d. Stomatitis

183.Irritation of the lips associated with scaling, crust formation, and fissures is termed

a. leukoplakia.
b. lichen planus.
c. actinic cheilitis.
d. chancre.

184.Regarding oral cancer, the nurse provides health teaching to inform the patient that

a. many oral cancers produce no symptoms in the early stages.
b. most oral cancers are painful at the outset.
c. Blood testing is used to diagnose oral cancer.
d.a typical lesion is soft and crater-like.

185.The most common symptom of esophageal disease is

a. dysphagia.
b. nausea.
c. vomiting.
d. odynophagia.

186.Halitosis and a sour taste in the mouth are signs and symptoms associated most directly with

a. esophageal diverticula.
b. achalasia.
c. gastroesophageal reflux.
d. hiatal hernia.

187.Which of the following terms refers to the symptom of gastroesophageal reflux disease (GERD) which is characterized by a burning sensation in the esophagus?

a. Pyrosis
b. Dyspepsia
c. Dysphagia
d. Odynophagia



188.The nurse teaches the patient with gastroesophageal reflux disease (GERD) which of the following measures to manage his disease?

a. Avoid eating or drinking 2 hours before bedtime.
b. Minimize intake of caffeine, beer, milk, and foods containing peppermint and spearmint.
c. Elevate the foot of the bed on 6- to 8-inch blocks
d. Eat a low carbohydrate diet

189.Which of the following statements accurately describes cancer of the esophagus?

a. Chronic irritation of the esophagus is a known risk factor.
b. It is three times more common in women in the U.S. than men.
c. It is seen more frequently in Caucasians than in African Americans.

d. It usually occurs in the fourth decade of life.

190. Which of the following venous access devices can be used for no more than 30 days in patients requiring parenteral nutrition?

a. Non-tunneled catheter
b. Peripherally-inserted central catheter (PICC)
c. Tunneled catheters
d. Implanted ports

191.To ensure patency of central venous line ports, dilute heparin flushes are used in which of the following situations?

a. Daily when not in use
b. With continuous infusions
c. Before blood drawing
d. When the line is discontinued

192.For which of the following medications must the nurse contact the pharmacist in consultation when the patient receives all oral medications by feeding tube?

a. Enteric-coated tablets
b. Simple compressed tablets
c. Buccal or sublingual tablets
d. Soft gelatin capsules filled with liquid

193.Medium -length nasoenteric tubes are used for:

a. Feeding
b. Decompression
c. Aspiration
d. Emptying

194.Mercury is typically used in the placement of which of the following tubes?

a. Miller-Abbott
b. Gastric sump
c. Dobbhoff
d.EnterafloW

195.The most significant nursing problem related to continuous tube feedings is

a. potential for aspiration
b. interruption of GI integrity
c. disturbance in the sequence of intestinal and hepatic metabolism
d. interruption in fat metabolism and lipoprotein synthesis

196.When the nurse prepares to give a bolus tube feeding to the patient and determines that the residual gastric content is 150 cc, her best action is to

a. reassess the residual gastric content in 1 hour.
b. notify the physician.
c. give the tube feeding.
d. withhold the tube feeding indefinitely.

197. If tube feeding is continuous, the placement of the feeding tube should be checked

a. every shift.
b. every hour.
c. every 24 hours.
d. when a tube feeding is continuous, it is unnecessary to check placement.

198.Decrease in absorption of which of the following vitamins in the geriatric patient results in pernicious anemia?

a. B12
b. C
c. D
d. B6

199.Which of the following terms refers to tarry, black stools?

a. Melena
b. Hemarthrosis
c.Hematemesis
d. Pyrosis

200. Which of the following statements accurately reflects a rule of thumb upon which the nurse may rely in assessing the patient's fluid balance?

a. Minimal intake of 1.5 liters per day
b. Minimal urine output of 50 milliliters per hour
c. Minimal urine output of 10 milliliters per hour
d. Minimal intake of 2 liters per day

201.The nurse recognizes that the patient with a duodenal ulcer will likely experience

a. pain 2-3 hours after a meal.
b. vomiting.
c. hemorrhage.
d. weight loss.

202. Of the following categories of medications, which is used in combination with bismuth salts to eradicate Helicobacter pylori?

a. Antibiotics
b. Antacids
c. Proton pump inhibitors
d. Histamine-2 receptor antagonists

203. Which of the following medications represents the category proton (gastric acid) pump inhibitors?

a. Omeprazole (Prilosec)
b. Sucralfate (Carafate)
c. Famotidine (Pepcid)
d. Metronidazole (Flagyl)

204. Which of the following medications used for the treatment of obesity prevents the absorption of triglycerides?

a. Orlistat (Xenical)
b. bupropion hydrochloride (Wellbutrin)
c. Sibutramine hydrochoride (Meridia)
d. Fluoxetine hydrochloride (Prozac)



205. Of the following bariatric surgical procedures, which is the best procedure for long-term weight loss?

a. Roux-en-Y
b. Vertical banded gastroplasty
c.Gastric ring application
d. Jejuno-ileal bypass

206. Which of the following statements regarding gastric cancer is accurate?

a. The incidence of cancer of the stomach continues to decrease in the United States.
b. Most gastric cancer deaths occur in people younger than 40 years.
c. Females have a higher incidence of gastric cancers than males.
d. A diet high in smoked foods and low in fruits and vegetables may decrease the risk of gastric cancer.

207. Which of the following categories of laxatives draw water into the intestines by osmosis?

a. Saline agents (milk of magnesia)
b. Bulk-forming agents (Metamucil)
c. Stimulants (Dulcolax)
d. Fecal softeners (Colace)

208.Crohn's disease is a condition of malabsorption caused by

a. inflammation of all layers of intestinal mucosa.
b. infectious disease.
c. disaccharidase deficiency.
d. gastric resection.


209.The nurse teaches the patient whose surgery will result in a sigmoid colostomy that the feces expelled through the colostomy will be

a. solid.
b. semi-mushy.
c. mushy.
d. fluid.

210. When irrigating a colostomy, the nurse lubricates the catheter and gently inserts it into the stoma no more than _______ inches

a. 3”
b. 2”
c. 4”
d. 5”

211.A longitudinal tear or ulceration in the lining of the anal canal is termed a (an)

a. anal fissure.
b. anorectal abscess.
c. anal fistula.
d. hemorrhoid.

212. Which type of diarrhea is caused by increased production and secretion of water and electrolyes by the intestinal mucosa into the intestinal lumen?

a. Secretory diarrhea
b. Osmotic diarrhea
c. Mixed diarrhea
d. Diarrheal disease

213.Which of the following terms is used to refer to intestinal rumbling?

a. Borborygmus
b. Tenesmus
c. Azotorrhea
d. Diverticulitis

214.The presence of mucus and pus in the stools suggests

a. Inflammatory colitis
b. Small bowel disease
c. Disorders of the colon
d. Intestinal malabsorption

215. Celiac sprue is an example of which category of malabsorption?

a. Mucosal disorders causing generalized malabsorption
b. Infectious diseases causing generalized malabsorption
c. Luminal problems causing malabsorption
d. Postoperative malabsorption

216.Typical signs and symptoms of appendicitis include:

a. Nausea
b. Left lower quadrant pain
c. Pain when pressure is applied to the right lower quadrant of the abdomen.
d. High fever

217.Regional enteritis is characterized by:

a. Transmural thickening
b. Diffuse involvement
c. Severe diarrhea
d. Exacerbations and remissions


218. What is the most common cause of small bowel obstruction?

a. Adhesions
b. Hernias
c. Neoplasms
d. Volvulus

219. Which of the follow statements provides accurate information regarding cancer of the colon and rectum?

a. Cancer of the colon and rectum is the second most common type of internal cancer in the United States.
b. Rectal cancer affects more than twice as many people as colon cancer.
c.The incidence of colon and rectal cancer decreases with age.
d. There is no hereditary component to colon cancer.

220. Which of the following characteristics are risk factors for colorectal cancer?

a. Familial polyposis
b. Age younger than 40
c. Low fat, low protein, high fiber diet
d. History of skin cancer

221.Which type of jaundice in adults is the result of increased destruction of red blood cells?

a. Hemolytic
b. Hepatocellular
c. Obstructive
d. Non-obstructive

222.The nurse places the patient after liver biopsy in which of the following
positions?

a. On the right side
b. On the left side
c. Trendelenburg
d. High Fowler's

223. Which of the following terms is used to describe a chronic liver disease in which scar tissue surrounds the portal areas?

a. Alcoholic cirrhosis
b. Postnecrotic cirrhosis
c. Biliary cirrhosis
d. Compensated cirrhosis

224.Which of the following terms describes the passage of a hollow instrument into a cavity for the withdrawal of fluid?

a. Paracentesis
b. Astrerixis
c. Ascites
d. Dialysis

225.Which of the following terms most precisely refers to the incision of the common bile duct for removal of stones?

a. Choledocholithotomy
b. Cholecystostomy
c. Choledochotomy
d. Choledochoduodenostomy

226.Which of the following clinical characteristics is associated with Type 1 diabetes (previously referred to as insulin-dependent diabetes mellitus [IDDM])?

a. Presence of islet cell antibodies
b. Obesity
c. Rare ketosis
d. Requirement for oral hypoglycemic agents

227.Which of the following clinical characteristics is associated with Type 2 diabetes (previously referred to as non-insulin-dependent diabetes mellitus [NIDDM])?

a. Can control blood glucose through diet and exercise
b. Usually thin at diagnosis
c. Ketosis-prone
d. Demonstrate islet cell antibodies

228. Of the following types of insulin, which is the most rapid acting?

a. Humalog
b. Regular
c. NPH
d. Ultralente

229. Of the following categories of oral antidiabetic agents, which exert their primary action by directly stimulating the pancreas to secrete insulin?

a. Sulfonylureas
b. Thiazolidinediones
c. Biguanides

d. Alpha glucosidase inhibitors

230.The nurse teaches the patient about diabetes including which of the following statements?

a. Elevated blood glucose levels contribute to complications of diabetes, such as
b. Sugar is found only in dessert foods.
c. The only diet change needed in the treatment of diabetes is to stop eating sugar.
d. Once insulin injections are started in the treatment of Type 2 diabetes, they can never be discontinued.

231.The nurse teaches the patient about glargine (Lantus), a "peakless" basal insulin including which of the following statements?

a. Do not mix the drug with other insulins
b. Administer the total daily dosage in two doses.
c. Draw up the drug first, then add regular insulin.
d. The drug is rapidly absorbed and has a fast onset of action.

232.Which of the following disorders is characterized by a group of symptoms produced by an excess of free circulating cortisol from the adrenal cortex?

a. Cushing's syndrome
b. Addison's disease
c. Graves' disease
d. Hashimoto's disease

233. Of the following disorders, which results from excessive secretion of somatotropin?

a. Acromegaly
b. Cretinism
c. Dwarfism

d. Adrenogenital syndrome

234.Which of the following hormones is secreted by the posterior pituitary?

a. Vasopressin
b. Calcitonin
c. Corticosteroids
d. Somatostatin

235.Trousseau's sign is positive when

a. carpopedal spasm is induced by occluding the blood flow to the arm for 3 minutes with the use of a blood pressure cuff.
b. a sharp tapping over the facial nerve just in front of the parotid gland and anterior to the ear causes spasm or twitching of the mouth, nose, and eye.
c. after making a clenched fist, the palm remains blanched when pressure is placed over the radial artery.
d. The patient complains of pain in the calf when his foot is dorsiflexed.

236.The digestion of carbohydrates is aided by

a. amylase.
b. lipase.
c. trypsin.
d. secretin.

237.The term used to describe total urine output of less than 400 mL in 24 hours is

a. oliguria.
b. anuria.
c. nocturia.
d. dysuria.


238.When fluid intake is normal, the specific gravity of urine should be

a. 1.010-1.025.
b. 1.000.
c. less than 1.010.
d. greater than 1.025.

239. Of the following terms, which refers to casts in the urine?

a. Cylindruria
b. Crystalluria
c. Pyuria
d. Bacteriuria

240. When the nurse observes the patient's urine to be orange, she further assesses the patient for

a. intake of medication such as phenytoin (Dilantin).
b. bleeding.
c. intake of multiple vitamin preparations.
d. infection.

241.To assess circulating oxygen levels the 2001 Kidney Disease Outcomes Quality Initiative: Management of Anemia Guidelines recommends the use of which of the following diagnostic tests?

a. Hemoglobin
b. Hematocrit
c. Serum iron levels
d. Arterial blood gases




242.Which of the following types of incontinence refers to involuntary loss of urine through an intact urethra as a result of a sudden increase in intra-abdominal pressure?

a. Stress
b. Overflow
c. Urge
d. Reflex

243.To facilitate entry of a catheter into the male urethra, the penis should be positioned at which of the following degree angles (in relation to the body)?

a. 90 degrees
b. 45 degrees
c. 180 degrees
d. 270 degrees

244.In assessing the appropriateness of removing a suprapubic catheter, the nurse recognizes that the patient's residual urine must be less than which of the following amounts on two separate occasions (morning and evening)?

a. 100 cc
b. 30 cc
c. 50 cc
d. 400 cc

245.When providing care to the patient with bilateral nephrostomy tubes, the nurse never does which of the following?

a. Clamps each nephrostomy tube when the patient is moved
b. Reports a dislodged nephrostomy tube immediately
c. Measures urine output from each tube separately
d. Irrigates each nephrostomy tube with 30 cc of normal saline q8h as ordered


246. Which type of medication may be used in the treatment of a patient with incontinence to inhibit contraction of the bladder?

a. Anticholinergic agent
b. Estrogen hormone
c. Tricyclic antidepressants
d. Over-the-counter decongestant

247.Which of the following is a reversible cause of urinary incontinence in the older adult?

a. Constipation.
b. Increased fluid intake
c. Age
d. Decreased progesterone level in the menopausal woman.

248.Bladder retraining following removal of an indwelling catheter begins with
instructing the patient to follow a 2-3 hour timed voiding schedule.
Immediately after the removal of the indwelling catheter, the patient is placed on a timed voiding schedule, usually two to three hours. At the given time interval, the patient is instructed to void.

a. encouraging the patient to void immediately.
b. advising the patient to avoid urinating for at least 6 hours.
c. performing straight catherization after 4 hours.

249. Which of the following terms is used to refer to inflammation of the renal pelvis?

a.Pyelonephritis
b. Cystitis
c. Urethritis
d. Interstitial nephritis


250.If an indwelling catheter is necessary, nursing interventions that should be implemented to prevent infection include

a. performing meticulous perineal care daily with soap and water.
b. using clean technique during insertion.
c. using sterile technique to disconnect the catheter from tubing to obtain urine specimens.
d. placing the catheter bag on the patient's abdomen when moving the patient.

251.The nurse who provides teaching to the female patient regarding prevention of recurrent urinary tract infections includes which of the following statements?

a. Void immediately after sexual intercourse.
b. Take tub baths instead of showers.
c. Increase intake of coffee, tea, and colas.
d. Void every 5 hours during the day.

252.A history of infection specifically caused by group A beta-hemolytic streptococci is associated with which of the following disorders?

a. Acute glomerulonephritis
b. Acute renal failure
c. Chronic renal failure
d. Nephrotic syndrome

253.Rejection of a transplanted kidney within 24 hours after transplant is termed

a. hyperacute rejection.
b. acute rejection.
c. chronic rejection.
d. simple rejection.


254.When caring for a patient with an uncomplicated, mild urinary tract infection (UTI), the nurse knows that recent studies have shown which of the following drugs to be a good choice for short-course (e.g. 3-day) therapy?

a. Levofloxacin (Levaquin)
b. Trimethoprim sulfamethoxazole (TMP-SMZ, Bactrim, Septra)
c. Nitrofurantoin (Macrodantin, Furadantin)
d. Ciprofloxacin (Cipro)

255.Which of the following terms refers to difficult or painful sexual intercourse?

a. Dyspareunia
b. Amenorrhea
c. Dysmenorrhea
d. Endometriosis

256.The opening into the vagina on the perineum is termed the

a. introitus.
b. adnexa.
c. cervix.
d. hymen.

257. Which of the following hormones is primarily responsible for stimulating the production of progesterone?

a. Luteinizing hormone
b. Follicle-stimulating hormone
c.Estrogen.
d. Androgen




258. When the results of a Pap smear are reported as class 5, the nurse recognizes that the common interpretation is

a. malignant.
b. normal.
c. probably normal.
d. suspicious.

259. For women aged 19-39 years, recommended health screening diagnostic testing includes which of the following?

a. Pap smear
b. Mammography
c. Cholesterol and lipid profile
d. Bone mineral density testing

260. Which of the following statements reflects nursing care of the woman with mild to moderate ovarian hyperstimulation syndrome (OHSS)?

a. Advise the patient to decrease her activity, monitor her urine output and to return for frequent office visits.
b. Advise the patient to measure her weight and abdominal circumference daily.
c. Advise the patient to monitor her heart rate and to report if her pulse falls below 60 beats per minute.
d. Prepare the patient for immediate hospitalization.

261. Which of the following terms is used to describe a procedure in which cervical tissue is removed as result of detection of abnormal cells?

a. Conization
b. Colporrhaphy
c. Cryotherapy
d. Perineorrhaphy


262. Of the following terms, which is used to refer to a type of gestational trophoblastic neoplasm?

a. Hydatidiform mole
b. Dermoid cyst
c. Doderlein's bacilli
d. Bartholin's cyst

263. When the female client reports a frothy yellow-brown vaginal discharge, the nurse suspects the client has a vaginal infection caused by

a. trichomonas vaginalis.
b. candida albicans.
c. gardnerella vaginalis.
d. chlamydia.

264. The nurse providing education regarding sexually transmitted diseases includes which of the following statements regarding herpes virus 2 (herpes genitalis)?

a. In pregnant women with active herpes virus, babies delivered vaginally may become infected with the virus.
b. Transmission of the virus requires sexual contact.
c. Transmission occurs only when the carrier has symptoms.
d. The virus is very difficult to kill.

265.An opening between the bladder and the vagina is called a

a. vesicovaginal fistula.
b. cystocele.
c. rectocele.
d. rectovaginal fistula.


266. Which of the following statements defines laparoscopic myomectomy—an alternative to hysterectomy for the treatment of excessive bleeding due to fibroids?

a.Removal of fibroids through a laparoscope inserted through a small abdominal incision.
b. Cauterization and shrinking of fibroids using a laser or electrical needles.
c. Coagulation of the fibroids using electrical current.
d. Resection of the fibroids using a laser through a hyserscope passed through the cervix.

267. Stage 3 of breast development, according to Tanner, occurs when

a. the areola (a darker tissue ring around the nipple) develops.
b. breast budding begins.
c. the areola and nipple form a secondary mound on top of breast tissue.
d. the breast develops into a single contour
In stage 5, the female demonstrates continued development of a larger breast with a single contour.

268. When the female patient demonstrates thickening, scaling, and erosion of the nipple and areola, the nurse recognizes that the patient is exhibiting signs of

a. Paget's disease.
b. acute mastitis.
c. fibroadenoma.
d. peau d'orange (edema).

269.The nurse teaches the female patient who is premenopausal to perform breast self-examination (BSE)

a. on day 5 to day 7, counting the first day of menses as day 1.
b. with the onset of menstruation
c. on day 2 to day 4, counting the first day of menses as day 1.

d. any time during the month.

270. Which type of biopsy is used for nonpalpable lesions found on mammography?

a. Stereotactic.
b. Excisional
c. Incisional.
d. Tru-Cut core

271.The nurse recognizes which of the following statements as accurately reflecting a risk factor for breast cancer?

a. Mother affected by cancer before 60 years of age
b. Onset of menses before 14 years of age
c. Multiparity
d. No alcohol consumption.

272. Which of the following terms is used to describe removal of the breast tissue and an axillary lymph node dissection leaving muscular structure intact as surgical treatment of breast cancer?

a. Modified radical mastectomy
b. Segmental mastectomy
c. Total mastectomy
d. Radical mastectomy

273. Ductal lavage is used for

a. women at higher risk for benign proliferative breast disease.
b. women at low risk for breast cancer.
c. screening women over age 65.
d. women with breast implants.


274. The 2000 NIH Consensus Development Conference Statement states that what percentage of women with invasive breast cancer should consider the option of systemic chemotherapy, not just women whose tumors are greater than 1cm in size?

a. 100% (all)
b. 75%
c. 50%
d. 25%

275. Which of the following terms refers to surgical removal of one of the testes?

a. Orchiectomy
b. Circumcision
c. Vasectomy
d. Hydrocelectomy

276. The term or disease associated with buildup of fibrous plaques in the sheath of the corpus cavernosum causing curvature of the penis when it is erect is known as

a. Peyronie's disease.
b. Bowen's disease.
c. phimosis.
d. priapism.

277. Which of the following terms is used to describe the opening of the urethra on the dorsum of the penis?

a. Epispadias
b. Hypospadias
c. Urethral stricture
d. Urethritis



278. The nurse teaches the patient who has been prescribed Viagra which of the following guidelines?

a. Do not take more than one tablet per day of your prescribed dose.
b. Viagra should be taken immediately before intercourse.
c. Viagra will result in erection formation.
d. Viagra will restore sex drive.

279. The obstructive and irritative symptom complex caused by benign prostatic hypertrophy is termed

a. prostatism.
b. prostatitis.
c. prostaglandin.
c. prostatectomy.

280. Proteins formed when cells are exposed to viral or foreign agents that are capable of activating other components of the immune system are referred to as

a. interferons.
b. antibodies.
c. antigens.
d. complements.

281.Cytotoxic T cells

a. lyse cells infected with virus.
b. are important in producing circulating antibodies.
c. attack foreign invaders (antigens) directly.
d. decrease B cell activity to a level at which the immune system is compatible with life.



282. During which stage of the immune response does the circulating lymphocyte containing the antigenic message return to the nearest lymph node?

a. Proliferation
b. Recognition
c. Response
d. Effector

283.Which of the following responses identifies a role of T lymphocytes?

a.Transplant rejection
b. Anaphylaxis
c. Allergic hay fever and asthma
d. Bacterial phagocytosis and lysis

284. Of the following classifications of medications, which is known to inhibit prostaglandin synthesis or release?

a. Nonsteroidal anti-inflammatory drugs (NSAIDs) in large doses
b. Antibiotics (in large doses)
c. Adrenal corticosteroids
d. Antineoplastic agents

285. Which of the following statements reflect current stem cell research?

a. The stem cell is known as a precursor cell that continually replenishes the body's entire supply of both red and white cells.
b. Stem cell transplantation can restore immune system functioning.
c. Stem cell transplantion has been performed in the laboratory only.
d. Clinical trials are underway in patients with acquired immune deficiencies only.



286. The nurse's base knowledge of primary immunodeficiencies includes which of the following statements? Primary immunodeficiencies

a. develop early in life after protection from maternal antibodies decreases.
b. occur most commonly in the aged population.
c. develop as a result of treatment with antineoplastic agents.
d. disappear with age.

287. Agammaglobulinemia is also known as

a. Bruton's disease.
b. Nezelof syndrome.
c. Wiskott-Aldrich syndrome.
d. Common variable immunodeficiency (CVID)

288. When the nurse administers intravenous gamma-globulin infusion, she recognizes that which of the following complaints, if reported by the patient, may indicate an adverse effect of the infusion?

a. Tightness in the chest
b. Nasal stuffiness
c. Increased thirst
d. Burning urination

289. Ataxia is the term that refers to

a. uncoordinated muscle movement.
b. vascular lesions caused by dilated blood vessels.
c. inability to understand the spoken word.
d. difficulty swallowing.



290. Which of the following microorganisms is known to cause retinitis in people with HIV/AIDS?

a. Cytomegalovirus
b. Cryptococcus neoformans
c.Mycobacterium avium.
d. Pneumocystic carinii

291. Of the following blood tests, which confirms the presence of antibodies to HIV?

a. Enzyme-linked immunoabsorbant assay (ELISA)
b. Erythrocyte sedimentation rate (ESR)
c. p24 antigen
d. Reverse transcriptase

292. When assisting the patient to interpret a negative HIV test result, the nurse informs the patient that the results mean.

a. his body has not produced antibodies to the AIDS virus.
b. he has not been infected with HIV.
c. he is immune to the AIDS virus.
d. antibodies to the AIDS virus are in his blood.

293. Which of the following substances may be used to lubricate a condom?

a. K-Y jelly
b. Skin lotion
c. Baby oil
d. Petroleum jelly

294. More than 500 CD4+ T lymphocytes/mm3 indicates which stage of HIV infection?

a. CDC category A - HIV asymptomatic
b. Primary infection (acute HIV infection or acute HIV syndrome)
c. CDC category B - HIV symptomatic
d. CDC category C - AIDS

295. The term used to define the balance between the amount of HIV in the body and the immune response is

a. viral set point
b. window period
c. primary infection stage
d. viral clearance rate

296. Which of the following statements reflect the treatment of HIV infection?

a. Treatment of HIV infection for an individual patient is based on the clinical condition of the patient, CD4 T cell count level, and HIV RNA (viral load).
b. Treatment should be offered to all patients once they reach CDC category B - HIV symptomatic.
c. Treatment should be offered to only selected patients once they reach CDC category B - HIV symptomatic.
d. Treatment should be offered to individuals with plasma HIV RNA levels less than 55,000 copies/mL (RT-PCR assay.)

297. Which of the following body substances causes increased gastric secretion, dilation of capillaries, and constriction of the bronchial smooth muscle?

a. Histamine
b. Bradykinin
c. Serotonin
Serotonin is a chemical mediator that acts as a potent vasoconstrictor and d. Prostaglandin


298. Which type of hypersensitivity reaction involves immune complexes formed when antigens bind to antibodies?

a. Type III
b. Type I
c. Type II
d. Type IV

299. When the patient's eosinophil count is 50-90% of blood leukocytes, the nurse interprets the result as

a. indicative of idiopathic hypereosinophilic syndrome.
b. indicating an allergic disorder.
c. suggesting an allergic reaction.
d. normal.

300. Which of the following interventions is the single most important aspect for the patient at risk for anaphylaxis?

a. Prevention
b. Use of antihistamines
c. Desensitization
d. Wearing of medical alert bracelet

301. When the nurse observes diffuse swelling involving the deeper skin layers in the patient who has experienced an allergic reaction, the nurse records the finding as

a. angioneurotic edema.
b. urticaria.
c. contact dermatitis.
d. pitting edema.


302. Atopic allergic disorders are characterized by

a. a hereditary predisposition.
b. an IgA-mediated reaction.
c. production of a systemic reaction.
d. a response to physiologic allergens.

303. The nurse teaches the patient with allergies about anaphylaxis including which of the following statements?

a. The most common cause of anaphylaxis is penicillin.
b. Anaphylactoid (anaphylaxis-like) reactions are commonly fatal.
c. The most common food item causing anaphylaxis is chocolate.
d. Systemic reactions include urticaria and angioedema

304. Which of the following statements describes the clinical manifestations of a delayed hypersensitivity (type IV) allergic reaction to latex?

a. Signs and symptoms are localized to the area of exposure, usually the back of the hands.
b. Signs and symptoms can be eliminated by changing glove brands or using powder-free gloves.
c. Signs and symptoms may worsen when hand lotion is applied before donning latex gloves.
d. Signs and symptoms occur within minutes after exposure to latex.

305. Which of the following terms refers to fixation or immobility of a joint?
a. Ankylosis
b. Hemarthrosis
c. Diarthrodial
d. Arthroplasty



306. Accumulation of crystalline depositions in articular surfaces, bones, soft tissue, and cartilage is referred to as

a. tophi.
b. subchondral bone.
c. pannus.
d. joint effusion.

307. Passive range-of-motion exercises are indicated during which stage of rheumatic disease?

a. Acute
b. Subacute
c. Inactive
d. Remission

308. Which of the following connective tissue disorders is characterized by insoluble collagen being formed and accumulating excessively in the tissues?

a. Scleroderma
b. Rheumatoid arthritis
c. Systemic lupus erythematosus
d. Polymyalgia rheumatic

309. Osteoarthritis is known as a disease that

a. is the most common and frequently disabling of joint disorders.
b. affects young males.
c. requires early treatment because most of the damage appears to occur early in the course of the disease.
d. affects the cartilaginous joints of the spine and surrounding tissues.


310. Which of the following newer pharmacological therapies used for the treatment of osteoarthritis is thought to improve cartilage function and retard degradation as well as have some anti-inflammatory effects?

a. Viscosupplementation
b. Glucosamine
c. Chondroitin
Chondroitin and glucosamine are thought to improve tissue function and retard d. Capsaicin

311. Which of the following statements reflect nursing interventions in the care of the patient with osteoarthritis?

a. Encourage weight loss and an increase in aerobic activity.
b. Provide an analgesic after exercise.
c. Assess for the gastrointestinal complications associated with COX-2 inhibitors.
d. Avoid the use of topical analgesics.

312. Fibromyalgia is a common condition that

a. involves chronic fatigue, generalized muscle aching and stiffness.
b. is caused by a virus.
c. is treated by diet, exercise, and physical therapy.
d. usually lasts for less than two weeks,

313. Which of the following terms refers to a condition characterized by destruction of the melanocytes in circumscribed areas of the skin?

a. Vitiligo
b. Hirsutism
c. Lichenification
d. Telangiectases


314. Of the following types of cells, which are believed to play a significant role in cutaneous immune system reactions?

a. Langerhans' cells
b. Merkel's cells
c. Melanocytes
d. Phagocytes

315. When the nurse assesses the patient and observes blue-red and dark brown plaques and nodules, she recognizes that these manifestations are associated with

a.Kaposi's sarcoma.
b. platelet disorders.
c. allergic reactions.
d. syphilis.

316.The nurse reading the physician's report of an elderly patient’s physical examination knows a notation that the patient demonstrates xanthelasma refers to

a. yellowish waxy deposits on upper eyelids.
b. liver spots.
c. dark discoloration of the skin.
d. bright red moles.

317.The nurse notes that the patient demonstrates generalized pallor and recognizes that this finding may be indicative of

a. anemia.
b. albinism.
c. vitiligo.
d. local arterial insufficiency.


318. Which of the following terms refers most precisely to a localized skin infection of a single hair follicle?

a. Furuncle
b. Carbuncle
c. Chelitis.
d. Comedone.

319. The nurse recommends which of the following types of therapeutic baths for its antipruritic action?

a. Colloidal (Aveeno, oatmeal)
b. Sodium bicarbonate (baking soda)
c. Water
d. Saline

320. Which of the following materials consists of a powder in water?

a. Suspension
b. Hygroscopic agent
c. Paste
d. Linament

321. Which of the following skin conditions is caused by staphylococci, streptococci, or multiple bacteria?

a. Impetigo
b. Scabies
c. Pediculosis capitis
d. Poison ivy



322. The nurse teaches the patient who demonstrates herpes zoster (shingles) that

a. the infection results from reactivation of the chickenpox virus.
b. once a patient has had shingles, they will not have it a second time.
c. a person who has had chickenpox can contract it again upon exposure to a person with shingles.
d. There are no known medications that affect the course of shingles.

323. Development of malignant melanoma is associated with which of the following risk factors?

a. Individuals with a history of severe sunburn
b. African-American heritage
c. People who tan easily
d. Elderly individuals residing in the Northeast

324. When caring for a patient receiving autolytic debridement therapy, the nurse

a. advises the patient about the foul odor that will occur during therapy.
b. ensures that the dressing is kept dry at all times.
c. ensures that the wound is kept open to the air for at least six hours per day.
d. Uses an enzymatic debriding agent such as Pancrease.

325. Which of the following reflect the pathophysiology of cutaneous signs of HIV disease?

a. Immune function deterioration
b. High CD4 count
c. Genetic predisposition
d. Decrease in normal skin flora




326. Most skin conditions related to HIV disease may be helped primarily by

a. highly active antiretroviral therapy (HAART).
b. symptomatic therapies.
c. low potency topical corticosteroid therapy.
d. improvement of the patient's nutritional status.

327. Which of the following terms refers to a graft derived from one part of a patient's body and used on another part of that same patient's body?

a. Autograft
b. Allograft
c. Homograft
d. Heterograft

328. When the emergency nurse learns that the patient suffered injury from a flash flame, the nurse anticipates which depth of burn?

a. Deep partial thickness
b. Superficial partial thickness
c. Full thickness
d. Superficial

329. Regarding emergency procedures at the burn scene, the nurse teaches which of the following guidelines?

a. Never wrap burn victims in ice.
b. Apply ice directly to a burn area.
c. Never apply water to a chemical burn.
d. Maintain cold dressings on a burn site at all times.




330. The first dressing change for an autografted area is performed

a. as soon as foul odor or purulent drainage is noted, or 3-5 days after surgery.
b. within 12 hours after surgery.
c. within 24 hours after surgery.
d. as soon as sanguineous drainage is noted.

331. Which of the following observations in the patient who has undergone allograft for treatment of burn site must be reported to the physician immediately?

a. Crackles in the lungs
b. Pain at the allograft donor site
c. Sanguineous drainage at the allograft donor site
d. Decreased pain at the allograft recipient site

332. Which of the following factors are associated with increased fluid requirements in the management of patients with burn injury?

a. Inhalation injuries
b. Chemical burn injuries
c. Low-voltage electrical injuries
d. Hypoglycemia

333. antimicrobial barrierActicoat dressings used in the treatment of burn wounds can be left in place for
a. five days.
b. seven to ten days.
c. three days.
d. two days.





334.A new biosynthetic dressing used , is used to treatin the treatment of burns, TransCyte

a. burns of indeterminate depth
b. partial-thickness burns.
c. superficial burns.
d. donor sites.

335. Which of the following statements reflect current research regarding the utilization of non-pharmacological measures in the management of burn pain?

a. Music therapy may provide reality orientation, distraction, and sensory stimulation.
b. Music therapy diverts the patient's attention toward painful stimulus.
c. Humor therapy has not proven effective in the management of burn pain.
d. Pet therapy has proven effective in the management of burn pain.

336. The most important intervention in the nutritional support of a patient with a burn injury is to provide adequate nutrition and calories to:

a. decrease catabolism.
b. increase metabolic rate.
c. increase glucose demands.
d. increase skeletal muscle breakdown.

337. Which of the following terms refers to the absence of the natural lens?

a. Aphakia
b. Scotoma
c. Keratoconus
d. Hyphema



338.Edema of the conjunctiva is termed

a. chemosis.
b. papilledema.
c. proptosis.
d. strabismus.

339. When the patient tells the nurse that his vision is 20/200, and asks what that means, the nurse informs the patient that a person with 20/200 vision

a. sees an object from 20 feet away that a person with normal vision sees from 200 feet away.
b. sees an object from 200 feet away that a person with normal vision sees from 20 feet away.
c. sees an object from 20 feet away that a person with normal vision sees from 20 feet away.
d. sees an object from 200 feet away that a person with normal vision sees from 200 feet away.

340. Which type of glaucoma presents an ocular emergency?

a. Acute angle-closure glaucoma
b. Normal tension glaucoma
c. Ocular hypertension
d. Chronic open-angle glaucoma

341. Which of the following categories of medications increases aqueous fluid outflow in the patient with glaucoma?

a. Cholinergics
b. Beta-blockers
c. Alpha-adrenergic agonists
d. Carbonic anhydrase inhibitors


342. Which of the following statements describe refractive surgery?

a. Refractive surgery is an elective, cosmetic surgery performed to reshape the cornea.
b. Refractive surgery will alter the normal aging of the eye.
c. Refractive surgery may be performed on all patients, even if they have underlying health conditions.
d. Refractive surgery may be performed on patients with an abnormal corneal structure as long as they have a stable refractive error.

343. The nurse knows that a postoperative vision-threatening complication of LASIK refractive surgery, diffuse lamellar keratitis (DLK) occurs

a. in the first week after surgery.
b. 1 month after surgery.
c. 2-3 months after surgery.
d. 6 months after surgery.

344. The nurse advises the patient undergoing photodynamic therapy (PDT) for macular degeneration to avoid exposure to direct sunlight or bright lights for

a. the first five days after the procedure.
b. the first 24 hours after the procedure.
c. two weeks after the procedure.
d. the first month after the procedure.

345. Retinoblastoma is the most common eye tumor of childhood; it is hereditary in

a. 30-40% of cases.
b. 10-20% of cases.
c. 25-50% of cases.
d. 50-75% of cases.


346. Which of the following terms refers to altered sensation of orientation in space?

a. Dizziness
b. Vertigo
c. Tinnitus
d. Nystagmus

347. Of the following terms, which describes a condition characterized by abnormal spongy bone formation around the stapes?

a. Otosclerosis
b. Middle ear effusion
c. Chronic otitis media
d. Otitis externa

348. Ossiculoplasty is defined as

a. surgical reconstruction of the middle ear bones.
b. surgical repair of the eardrum.
c. incision into the tympanic membrane.
d. incision into the eardrum.

349. Which of the following terms refers to surgical repair of the tympanic membrane?

a. Tympanoplasty
b. Tympanotomy
c. Myringotomy
d. Ossiculoplasty

350. Of the following tests, which uses a tuning fork between two positions to assess hearing?

a. Rinne's
b. Whisper
c. Watch tick
d. Weber's

351. Which of the following conditions of the inner ear is associated with normal hearing?

a.Vestibular neuronitis
b. Meniere's disease
c. Labyrinthitis
d. Endolymphatic hydrops

352. Of the following terms, which refers to the progressive hearing loss associated with aging?

a. Presbycusis
b. Exostoses
c. Otalgia
d. Sensorineural hearing loss

353. Which of the following statements describes benign paroxysmal positional vertigo (BPPV)?

a. The vertigo is usually accompanied by nausea and vomiting; however hearing impairment does not generally occur.
b.The onset of BPPV is gradual.
c. BPPV is caused by tympanic membrane infection.
d. BPPV is stimulated by the use of certain medication such as acetaminophen.

354. Nursing management of the patient with acute symptoms of benign paroxysmal positional vertigo includes which of the following?

a. Bed rest

b. The Epley repositioning procedure
c. Meclizine for 2-4 weeks
d. The Dix-Hallpike procedure.

355. Which of the following terms refers to the inability to recognize objects through a particular sensory system?

a. Agnosia
b. Dementia
c. Ataxia
d. Aphasia

356. Which of the following terms refers to weakness of both legs and the lower part of the trunk?

a. Paraparesis
b. Hemiplegia
c. Quadriparesis
d. Paraplegia

357. Of the following neurotransmitters, which demonstrates inhibitory action, helps control mood and sleep, and inhibits pain pathways?

a. Serotonin
b. Enkephalin
c. Norepinephrine
d. Acetylcholine

358. The lobe of the brain that contains the auditory receptive areas is the ____________ lobe.

a. temporal
b. frontal

c. parietal
d. occipital

359. The lobe of the brain that is the largest and controls abstract thought is the ____________ lobe.

a. frontal
b. temporal
c. parietal
d. occipital

360. Which of the following terms is used to describe the fibrous connective tissue that covers the brain and spinal cord?

a. Meninges
b. Dura mater
c. Arachnoid mater
d. Pia mater

361. The cranial nerve that is responsible for salivation, tearing, taste, and sensation in the ear is the _____________________ nerve.

a. vestibulocochlear
b. oculomotor
c. trigeminal
d. facial

362. The cranial nerve that is responsible for muscles that move the eye and lid is the _____________________ nerve.

a. oculomotor
b. trigeminal
c. vestibulocochlear

d. facial

363.The cranial nerve that is responsible for facial sensation and corneal reflex is the _____________________ nerve.

a. trigeminal
b. oculomotor
c. vestibulocochlear
d. facial

364. Upper motor neuron lesions cause

a. no muscle atrophy.
b. decreased muscle tone.
c. flaccid paralysis.
d. absent or decreased reflexes.

365. Lower motor neuron lesions cause

a. flaccid muscle paralysis.
b. increased muscle tone.
c. no muscle atrophy.
d. hyperactive and abnormal reflexes.

366. The percentage of patients over the age of 70 admitted to the hospital with delirium is about

a. 25%.
b. 10%.
c. 40%.
d. 50%.



367. Structural and motor changes related to aging that may be assessed in geriatric patients during an examination of neurologic function include which of the following?

a. Decreased or absent deep tendon reflexes
b. Increased pupillary responses
c. Increased autonomic nervous system responses.
d. Enhanced reaction and movement times

368. What safety actions does the nurse need to take for a patient on oxygen therapy who is undergoing magnetic resonance imaging (MRI)?

a. Ensure that no patient care equipment containing metal enters the room where the MRI is located.
b. Securely fasten the patient's portable oxygen tank to the bottom of the MRI table after the patient has been positioned on the top of the MRI table.
c. Check the patient's oxygen saturation level using a pulse oximeter after the patient has been placed on the MRI table.
d. No special safety actions need to be taken.

369. Which of the following terms refer to a method of recording, in graphic form, the electrical activity of the muscle?

a. Electromyogram
b. Electroencephalogram
c. Electrocardiography
d. Electrogastrography

370. Which of the following are sympathetic effects of the nervous system?

a. Dilated pupils
b. Decreased blood pressure
c. Increased peristalsis
d. Decreased respiratory rate

371. Lesions in the temporal lobe may result in which of the following types of agnosia?

a. Auditory
b. Visual
c. Tactile
d. Relationship

372. When the nurse observes that the patient has extension and external rotation of the arms and wrists and extension, plantar flexion, and internal rotation of the feet, she records the patient's posturing as

a. decerebrate.
b. normal.
c. flaccid.
d. decorticate.

373. Monro-Kellie hypothesis refers to

a. the dynamic equilibrium of cranial contents.
b. unresponsiveness to the environment.
c. the brain's attempt to restore blood flow by increasing arterial pressure to overcome the increased intracranial pressure.
d. a condition in which the patient is wakeful but devoid of conscious content, without cognitive or affective mental function.

374. A patient who demonstrates an obtunded level of consciousness

a. sleeps almost constantly but can be aroused and can follow simple commands.
b. has difficulty following commands, and may be agitated or irritable.
c. sleeps often and shows slowed speech and thought processes.
d. does not respond to environmental stimuli.


375. An osmotic diuretic, such as Mannitol, is given to the patient with increased intracranial pressure (IICP) in order to

a. dehydrate the brain and reduce cerebral edema.
b. control fever.
c. control shivering.
d. reduce cellular metabolic demands.

376. Which of the following positions are employed to help reduce intracranial pressure (ICP)?

a. Avoiding flexion of the neck with use of a cervical collar
b. Keeping the head flat with use of no pillow
c. Rotating the neck to the far right with neck support
d. Extreme hip flexion supported by pillows

377. Which of the following insults or abnormalities most commonly causes ischemic stroke?

a. Cocaine use
b. Arteriovenous malformation
c. Trauma
d. Intracerebral aneurysm rupture


378. When the patient is diagnosed as having global aphasia, the nurse recognizes that the patient will

a. be unable to form words that are understandable or comprehend the spoken word.
b. be unable to comprehend the spoken word.
c. be unable to form words that are understandable.
d. be unable to speak at all.


379. Which of the following terms related to aphasia refers to the inability to perform previously learned purposeful motor acts on a voluntary basis?

a. Apraxia
b. Agnosia
c. Agraphia
d. Perseveration

380. Which of the following terms related to aphasia refers to the failure to recognize familiar objects perceived by the senses?

a. Agnosia
b. Agraphia
c. Apraxia
d. Perseveration

381. Which of the following terms related to aphasia refers to difficulty reading?

a. Alexia
b. Agnosia
c. Agraphia
d. Perseveration

382. Which of the following terms related to aphasia refers to difficulty in selecting appropriate words, particularly nouns?

a. Anomia
b. Acalculia
c. Dysarthria
d. Paraphasia




383. A patient has had neurologic deficits lasting for more than 24 hours, and now the symptoms are resolving. The nurse concludes that the patient has had which type of stroke?

a. Reversible ischemic neurologic deficit
b. Transient ischemic attack (TIA)
c. Stroke in evolution
d. Completed stroke

384. Which of the following is a modifiable risk factor for transient ischemic attacks and ischemic strokes?

a. History of smoking.
b. Thyroid disease
c. Social drinking
d. Advanced age

385. A patient who has had a previous stroke and is taking warfarin tells the nurse that he started taking garlic to help reduce his blood pressure. The nurse knows that garlic when taken together with warfarin

a. can greatly increase the international normalization ratio (INR) and therefore increase the risk of bleeding.
b. have no drug-drug interactions and therefore may be taken together.
c. can cause platelet aggregation and therefore increase the risk of blood clotting.
d. may increase cerebral blood flow causing migraine headaches.

386. Later signs of increased intracranial pressure (ICP) later include which of the following?

a. Projectile vomiting
b. Increased pulse rate
c. Decreased blood pressure

d. Narrowed pulse pressure

387.Bleeding between the dura mater and arachnoid membrane is termed

a. subdural hematoma.
b. intracerebral hemorrhage.
c. epidural hematoma.
d. extradural hematoma.

388. Which of the following statements reflect nursing management of the patient with expressive aphasia?

a. Encourage the patient to repeat sounds of the alphabet.
b. Speak clearly and in simple sentences; use gestures or pictures when able.
c. Speak slowly and clearly to assist the patient in forming the sounds.
d. Frequently reorient the patient to time, place, and situation.

389. Health promotion efforts to decrease the risk for ischemic stroke involve encouraging a healthy lifestyle including

a. a low fat, low cholesterol diet, and increasing exercise.
b. eating fish no more than once a month.
c. a high protein diet and increasing weight-bearing exercise.
d. a low cholesterol, low protein diet, and decreasing aerobic exercise.

390. Before the patient diagnosed with a concussion is released from the Emergency Department, the nurse teaches the family or friends who will be tending to the patient to contact the physician or return to the ED if the patient

a. vomits.
b. complains of headache.
c. complains of generalized weakness.
d. sleeps for short periods of time.


391. When the nurse reviews the physician's progress notes for the patient who has sustained a head injury and sees that the physician observed Battle's sign when the patient was in the Emergency Department, the nurse knows that the physician observed

a. an area of bruising over the mastoid bone.
b. a bloodstain surrounded by a yellowish stain on the head dressing.
c. escape of cerebrospinal fluid (CSF) from the patient's ear.
d. escape of cerebrospinal fluid (CSF) from the patient's nose.

392. Which of the following findings in the patient who has sustained a head injury indicate increasing intracranial pressure (ICP)?

a. Widened pulse pressure
b. Increased pulse
c. Decreased respirations
d. Decreased body temperature

393. Which of the following nursing interventions is appropriate when caring for the awake and oriented head injury patient?

a. Supply oxygen therapy to keep blood gas values within normal range.
b. Do not elevate the head of the bed.
c. Encourage the patient to cough every 2 hours.
d. Use restraints if the patient becomes agitated.

394. Of the following stimuli, which is known to trigger an episode of autonomic hyperreflexia in the patient who has suffered a spinal cord injury?

a. Applying a blanket over the patient
b. Diarrhea
c. Placing the patient in a sitting position

d. Voiding

395. Risk factors that increase the likelihood of post-traumatic seizures following a head injury include which of the following?

a. Age over 65 years
b. Loss of consciousness for less than 1 day
c. Glasgow Coma Scale (GCS) score less than 10
d. Epidural hematoma

396. A post-traumatic seizure classified as early occurs

a. within 1-7 days of injury.
b. within 4 hours of injury.
c. within 24 hours of injury.
d. more than 7 days following surgery.

397. The nurse assesses the dressing of a patient with a basal skull fracture and sees the halo sign - a blood stain surrounded by a yellowish stain. The nurse knows that this sign

a. is highly suggestive of a cerebrospinal fluid (CSF) leak.
b. may indicate a subdural hematoma..
c. is highly suggestive of a cerebral contusion.
d. normally occurs within 24 hours following a basal skull fracture.

398. A Glasgow Coma Scale (GCS) score of 7 or less is generally interpreted as

a. coma.
b. a need for emergency attention.
c. least responsive.
d. most responsive.


399. Which of the following terms refers to muscular hypertonicity with increased resistance to stretch?

a. Spasticity
b. Akathesia
c. Ataxia
d. Myclonus

400. Of the following terms, which refers to blindness in the right or left halves of the visual fields of both eyes?

a.Homonymous hemianopsia
b. Scotoma
c. Diplopia
d. Nystagmus

401.Which of the following terms is used to describe rapid, jerky, involuntary, purposeless movements of the extremities?

a.Chorea
b. Bradykinesia
c. Dyskinesia
d. Spondylosis

402. Which of the phases of a migraine headache usually lasts less than an hour?

a. Aura
b.Prodrome
c.Headache
d. Recovery



403.The most common type of brain neoplasm is the

a. glioma.
b. angioma.

c. meningioma.

d. neuroma.

404. Which of the following diseases is a chronic, degenerative, progressive disease of the central nervous system characterized by the occurrence of small patches of demyelination in the brain and spinal cord?

a. Multiple sclerosis
b. Parkinson's disease
c. Huntington's disease
d. Creutzfeldt-Jakob's disease

405. Which of the following diseases is associated with decreased levels of dopamine due to destruction of pigmented neuronal cells in the substantia nigra in the basal ganglia of the brain?

a. Parkinson's disease
b. Multiple sclerosis
c. Huntington's disease
d. Creutzfeldt-Jakob's disease

406. Which of the following diseases is a chronic, progressive, hereditary disease of the nervous system that results in progressive involuntary dance-like movement and dementia?

a. Huntington's disease
b. Multiple sclerosis

c. Parkinson's disease
d. Creutzfeldt-Jakob's disease

407. Which of the following diseases is a rare, transmissible, progressive fatal disease of the central nervous system characterized by spongiform degeneration of the gray matter of the brain?

a. Creutzfeldt-Jakob's disease
b. Multiple sclerosis
c. Parkinson's disease
d. Huntington's disease

408. Bell's palsy is a disorder of which cranial nerve?

a. Facial (VII)
b. Trigeminal (V)
c. Vestibulocochlear (VIII)
d. Vagus (X)

409. The most common cause of acute encephalitis in the United States is

a. Herpes Simplex Virus (HSV).
b. Cryptococcus neoformans.
c. Western equine bacteria.
d. Candida albicans.

410. Which of the following reflects basic nursing measures in the care of the patient with viral encephalitis?

a. Providing comfort measures
b. Administering narcotic analgesics
c. Administering amphotericin B.
d. Monitoring cardiac output

411. Nursing management of the patient with new variant Creutzfeldt-Jakob Disease (nvCJD) includes

a. providing supportive care.
b. initiating isolation procedures.
c. preparing for organ donation.
d. administering amphotericin B.

412. Three medications referred to as the 'ABC drugs' are currently the main pharmacological therapy for multiple sclerosis. Which of the following statements reflects information to be included in patient teaching?

a. Flu-like symptoms can be controlled with nonsteroidal anti-inflammatory drugs (NSAIDs) and usually resolve after a few months of therapy.
b. Take interferon beta-la (Avonex) with food or milk.
c. Take interferon beta-1b (Betaseron) at night before bedtime for best effects.
d. Take glatiramer acetate (Copaxone) on an empty stomach.

413. Korsakoff's syndrome is characterized by

a. psychosis, disorientation, delirium, insomnia, and hallucinations.
b. severe dementia and myocLonus.
c. tremor, rigidity, and bradykinesia.
d. choreiform movement and dementia.

414. The primary North American vector transmitting arthropod-borne virus encephalitis is the

a. mosquito
b. tick.
c. horse.
d. flea.


415. The initial symptoms of new variant Creutzfeldt-Jakob Disease (nvCJD) are

a. anxiety, depression, and behavioral changes.
b. memory and cognitive impairment.
c. diplopia and bradykinesia.
d. akathisia and dysphagia.

416. A patient with fungal encephalitis receiving amphotericin B complaints of fever, chills, and body aches. The nurse knows that these symptoms

a. may be controlled by the administration of diphenhydramine (Benedryl) and acetaminophen (Tylenol) approximately 30 minutes prior to administration of the amphotericin.
b. indicate renal toxicity and a worsening of the patient's condition.
c. are primarily associated with infection with Coccidioides immitis and Aspergillus.
d. indicate the need for immediate blood and cerebral spinal fluid (CSF) cultures.

417.The patient with Herpes Simplex Virus (HSV) encephalitis is receiving acyclovir (Zovirax). The nurse monitors blood chemistry test results and urinary output for

a. renal complications related to acyclovir therapy.
b. signs and symptoms of cardiac insufficiency.
c. signs of relapse.
d. signs of improvement in the patient's condition.

418. Medical management of arthropod-borne virus (arboviral) encephalitis is aimed at

a. controlling seizures and increased intracranical pressure.
b. preventing renal insufficiency.
c. maintaining hemodynamic stability and adequate cardiac output.
d. preventing muscular atrophy.


419. The patient receiving mitoxantrone (Novantrone) for treatment of secondary progressive multiple sclerosis (MS) is closely monitored for

a. leukopenia and cardiac toxicity.
b. mood changes and fluid and electrolyte alterations.
c. renal insufficiency.
d. hypoxia.

420. What percentage of patients who survived the polio epidemic of the 1950s are now estimated to have developed post-polio syndrome?

a. 60-80%
b. 50%
c. 25-30%
d. 10%

421. Which of the following statements describe the pathophysiology of post-polio syndrome?

a. The exact cause is unknown, but aging or muscle overuse is suspected.
b. The exact cause is unknown, but latent poliovirus is suspected.
c. Post-polio syndrome is caused by an autoimmune response.
d. Post-polio syndrome is caused by long-term intake of a low-protein, high-fat diet in

422. Which of the following statements reflect nursing interventions of a patient with post-polio syndrome?

a. Providing care aimed at slowing the loss of strength and maintaining the physical, psychological and social well being of the patient.
b. Administering antiretroviral agents.
c. Planning activities for evening hours rather then morning hours.
d. Avoiding the use of heat applications in the treatment of muscle and joint pain.


423. Which of the following terms is used to describe edema of the optic nerve?

a. Papilledema
b. Scotoma
c. Lymphedema
d. Angioneurotic edema

424. Degenerative neurologic disorders include which of the following?

a. Huntington's disease
b. Paget's disease
c. Osteomyelitis
d. Glioma

425. Bone density testing in patients with post-polio syndrome has demonstrated

a. low bone mass and osteoporosis.
b. osteoarthritis.
c. calcification of long bones.
d. no significant findings.

426. Which of the following terms refers to mature compact bone structures that form concentric rings of bone matrix?

a. Lamellae
b. Endosteum
c.Trabecula
d. Cancellous bone




427.An osteon is defined as a

a. microscopic functional bone unit.
b. bone-forming cell.
c. bone resorption cell.
d. mature bone cell.

428. Which of the following terms refers to the shaft of the long bone?

a. Diaphysis
b. Epiphysis
c. Lordosis
d. Scoliosis

429. Paresthesia is the term used to refer to

a. abnormal sensations..
b. absence of muscle movement suggesting nerve damage.
c. involuntary twitch of muscle fibers.
d. absence of muscle tone.

430. Which of the following terms refers to a grating or crackling sound or sensation?

a. Crepitus
b. Callus
c. Clonus
d. Fasciculation

431. Which of the following terms refers to muscle tension being unchanged with muscle shortening and joint motion?

a. Isotonic contraction
b. Isometric contraction

c. Contracture
d. Fasciculation

432. During which stage or phase of bone healing after fracture does callus formation occur?

a. Reparative
b. Remodeling
c. Inflammation
d. Revascularization

433. During which stage or phase of bone healing after fracture is devitalized tissue removed and new bone reorganized into its former structural arrangement?

a. Remodeling
b. Inflammation
c. Revascularization
d. Reparative

434. Which nerve is assessed when the nurse asks the patient to spread all fingers?

a. Ulnar
b. Peroneal
c. Radial
d. Median

435. Which nerve is assessed when the nurse asks the patient to dorsiflex the ankle and extend the toes?

a. Peroneal
b. Radial
c. Median
d. Ulnar



436. Which of the following statements reflect the progress of bone healing?

a. Serial x-rays are used to monitor the progress of bone healing.
b. All fracture healing takes place at the same rate no matter the type of bone fractured.
c. The age of the patient influences the rate of fracture healing.
d. Adequate immobilization is essential until there is ultrasound evidence of bone formation with ossification.

437. Diminished range of motion, loss of flexibility, stiffness, and loss of height are history and physical findings associated with age-related changes of the

a. joints.
b. bones.
c. muscles.
d. ligaments.

438. Fracture healing occurs in four areas, including the

a. external soft tissue.
b. cartilage.
c. bursae.
d. fascia.

439. Which of the following is an indicator of neurovascular compromise?

a. Capillary refill more than 3 seconds
b. Warm skin temperature
c. Diminished pain
d. Pain on active stretch.


440. Which of the following terms refers to moving away from midline?

a. Abduction
b. Adduction
c. Inversion
d. Eversion

441.Surgical fusion of a joint is termed

a. arthrodesis.
b. open reduction with internal fixation (ORIF).
c. heterotrophic ossification.
d. arthroplasty.

442. Which of the following devices is designed specifically to support and immobilize a body part in a desired position?

a. Splint
b. Brace
c. Continuous passive motion (CPM) device
d. Trapeze

443. When caring for the patient in traction, the nurse is guided by which of the following principles?

a. Skeletal traction is never interrupted.
b. Weights should rest on the bed.
c. Knots in the ropes should touch the pulley.
d. Weights are removed routinely.

444. Meniscectomy refers to the

a. replacement of one of the articular surfaces of a joint.

b. incision and diversion of the muscle fascia.
c. excision of damaged joint fibrocartilage.
d. removal of a body part.

445. In order to avoid hip dislocation after replacement surgery, the nurse teaches the patient which of the following guidelines?

a. Never cross the affected leg when seated.
b. Keep the knees together at all times.
c. Avoid placing a pillow between the legs when sleeping.
d. Bend forward only when seated in a chair.

446. Injury to the ______ nerve as a result of pressure is a cause of footdrop.

a. Peroneal
b. Sciatic
c. Femoral
d. Achilles

447. The nurse teaching the patient with a cast about home care includes which of the following instructions?

a. Dry a wet fiberglass cast thoroughly using a hair dryer on a cool setting to avoid skin problems.
b. Cover the cast with plastic or rubber.
c. Keep the cast below heart level.
d. Fix a broken cast by applying tape.

448. A continuous passive motion (CPM) device applied after knee surgery

a. promotes healing by increasing circulation and movement of the knee joint.
b. provides active range of motion.
c. promotes healing by immobilizing the knee joint.

d. prevents infection and controls edema and bleeding.

449. Which of the following terms refers to disease of a nerve root?

a. Radiculopathy
b. Involucrum
c. Sequestrum
d. Contracture


450. Of the following common problems of the upper extremities, which results from entrapment of the median nerve at the wrist?

a. Carpal tunnel syndrome
b. Ganglion
c. Dupuytren's contracture
d. Impingement syndrome

451. When the nurse notes that the patient's left great toe deviates laterally, she recognizes that the patient has a

a. hallux valgus.
b. hammertoe.
c. pes cavus.
d. flatfoot.

452. Localized rapid bone turnover, most commonly affecting the skull, femur, tibia, pelvic bones, and vertebrae, characterizes which of the following bone disorders?

a. Osteitis deformans
b. Osteomalacia
c. Osteoporosis
d. Osteomyelitis


453. Most cases of osteomyelitis are caused by which of the following microorganisms?

a. Staphylococcus
b. Proteus species
c. Pseudomonas species
d. Escherichia coli

454. Which of the following statements reflects information to be included when teaching the patient about plantar fasciitis?

a. Management of plantar fasciitis includes stretching exercises.
b. Plantar fasciitis presents as an acute onset of pain localized to the ball of the foot that occurs when pressure is placed upon it and diminishes when pressure is released.
c. The pain of plantar fasciitis diminishes with warm water soaks.
d. Complications of plantar fasciitis include neuromuscular damage and decreased ankle range of motion.

455.Lifestyle risk factors for osteoporosis include

a. lack of exposure to sunshine.
b. lack of aerobic exercise.
c. a low protein, high fat diet.
d. an estrogen deficiency or menopause.

456. The nurse teaches the patient with a high risk for osteoporosis about risk-lowering strategies including which of the following statements?

a. Walk or perform weight-bearing exercises out of doors..
b. Increase fiber in the diet.
c. Reduce stress.
d. Decrease the intake of vitamin A and D.

457.Instructions for the patient with low back pain include which of the following?

a. When lifting, avoid overreaching.
b. When lifting, place the load away from the body.
c. When lifting, use a narrow base of support.
d. When lifting, bend the knees and loosen the abdominal muscles.

458. Dupuytren's contracture causes flexion of the

a. fourth and fifth fingers.
b. thumb.
c. index and middle fingers.
d. ring finger.

459. A metabolic bone disease characterized by inadequate mineralization of bone is

a. osteomalacia
b. osteoporosis
c. osteomyelitis
d. osteoarthritis

460. Which of the following terms refers to an injury to ligaments and other soft tissues of a joint?

a. Sprain
b. Dislocation
c. Subluxation
d. Strain

461. Which of the following terms refers to failure of fragments of a fractured bone to heal together?

a. Nonunion

b. Dislocation
c. Subluxation
d. Malunion

462. The Emergency Department nurse teaches patients with sports injuries to remember the acronym RICE, which stands for which of the following combinations of treatment?

a. Rest, ice, compression, elevation
b. Rest, ice, circulation, and examination
c.Rotation, immersion, compression and elevation
d. Rotation, ice, compression, and examination

463.The nurse anticipates that the physician will perform joint aspiration and wrapping with compression elastic dressing for which of the following musculoskeletal problems?

a. Joint effusion
b. Strain
c. Sprain
d. Avascular necrosis

464. When x-ray demonstrates a fracture in which bone has splintered into several pieces, that fracture is described as

a. comminuted.
b. compound.
c. depressed.
d. impacted.

465. When x-ray demonstrates a fracture in which the fragments of bone are driven inward, the fracture is described as

a. depressed.

b. compound.
c. comminuted.
d. impacted.

466. A fracture is termed pathologic when the fracture

a. occurs through an area of diseased bone.
b. results in a pulling away of a fragment of bone by a ligament or tendon and its attachment.
c. presents as one side of the bone being broken and the other side being bent.
d. involves damage to the skin or mucous membranes.

467. The most common complication after knee arthroscopy is

a. joint effusion.
b. infection.
c. knee giving way.
d. knee locking.

468. When the patient who has experienced trauma to an extremity complains of severe burning pain, vasomotor changes, and muscles spasms in the injured extremity, the nurse recognizes that the patient is likely demonstrating signs of

a. reflex sympathetic dystrophy syndrome.
b. avascular necrosis of bone.
c. a reaction to an internal fixation device.
d. heterotrophic ossification.

469. Which of the following terms refers to a fracture in which one side of a bone is broken and the other side is bent?

a. Greenstick

b. Spiral
c. Avulsion
d. Oblique

470. The nurse assesses subtle personality changes, restlessness, irritability, and confusion in a patient who has sustained a fracture. The nurse suspects

a. fat embolism syndrome.
b. compartment syndrome.
c. hypovolemic shock.
d. reflex sympathetic dystrophy syndrome.

471. A Colles' fracture is a fracture of the

a. distal radius.
b. elbow.
c. humeral shaft.
d. clavicle.

472. With fractures of the femoral neck, the leg is

a. shortened, adducted, and externally rotated.
b. shortened, abducted, and internally rotated.
c. adducted and internally rotated.
d. abducted and externally rotated.

473.Which of the following terms most precisely refers to an infection acquired in the hospital that was not present or incubating at the time of hospital admission?

a. Nosocomial infection
b. Primary bloodstream infection
c. Secondary bloodstream infection
d. Emerging infectious diseases



474.The usual incubation period (infection to first symptom) for AIDS is

a. 10 years.
b. 3--6 months.
c. 1 year.
d. 5 years.

475. The usual incubation period (infection to first symptom) for hepatitis B is

a. 45-160 days.
b. 15-50 days.
c. 6-9 months.
d. The incubation periods for hepatitis D, E, and G are unclear.


476. Which of the following terms refers to a state of microorganisms being present within a host without causing host interference or interaction?

a. Colonization
b. Susceptible
c. Immune
d. Infection

477. The nurse teaches the parent of the child with chickenpox that the child is no longer contagious to others when

a. the vesicles and pustules have crusted.
b. the first rash appears.
c. the fever disappears.
d. the rash is changing into vesicles, and pustules appear.


478. Which of the following statements reflects the nursing management of the patient with West Nile Virus infection?

a. There is no treatment for West Nile Virus infection.
b. The incubation period is three to five days.
c. Patients with West Nile virus present with gastrointestinal complaints, such as nausea, vomiting, diarrhea, and abdominal pain.
d. Transmission of West Nile virus occurs from human-to-human.

479. Prophylaxis antibiotic for anthrax is given to people with symptoms who have been in a defined "hot zone" for a period of

a. 60 days.
b. 30 days.
c. 14 days.
d. 10 days.

480. If a case of smallpox is suspected, the nurse should

a. call the CDC Emergency Preparedness Office.
b. immediately vaccinate the patient and anyone in contact with the patient.
c. establish isolation with positive pressure.
d. Assess the patient for signs of a rash similar to chickenpox in appearance and progression.

481. The six elements necessary for infection are a causative organism, a reservoir of available organisms, a portal or mode of exit from the reservoir, a mode of transmission from reservoir to host, a susceptible host, and a

a. mode of entry to host.
b. mode of exit from the host.
c. virulent host.
d. latent time period.

482. Which of the following statements reflect what is known about the Ebola and Marburg viruses?

a. The diagnosis should be considered in a patient who has a febrile, hemorrhagic illness after traveling to Asia or Africa.
b. Treatment during the acute phase includes administration of acyclovir, and ventilator and dialysis support.
c. The viruses can be spread only by airborne exposure.
d. Symptoms include severe lower abdominal pain, nausea, vomiting, and dehydration.

483.Bubonic plague occurs

a. after the organism enters through the skin.
b. occurs after the organism is inhaled..
c. occurs when the organism causes a bloodstream infection.
d. after the organism is transferred by human to human contact.

484.The term given to the category of triage that refers to life-threatening or potentially life-threatening injury or illness requiring immediate treatment is

a. emergent.
b. urgent.
c. immediate..
d. non-acute.

485.When the patient has been field triaged and categorized as blue, the nurse recognizes that the patient requires

a. fast-track or psychological support.
b. emergent care.
c. immediate care.
d. urgent care.


486. Which of the following guidelines is appropriate to helping family members cope with sudden death?

a. Show acceptance of the body by touching it, giving the family permission to touch.
b. Inform the family that the patient has passed on.
c. Obtain orders for sedation for family members.
d. Provide details of the factors attendant to the sudden death.

487. Which of the following solutions should the nurse anticipate for fluid replacement in the male patient?

a. Lactated Ringer's solution.
b. Type O negative blood
c. Dextrose 5% in water
d. Hypertonic saline

488.Induction of vomiting is indicated for the accidental poisoning patient who has ingested

a. aspirin.
b. rust remover.
c. gasoline.
d. toilet bowl cleaner.

489. Which of the following phases of psychological reaction to rape is characterized by fear and flashbacks?

a. Heightening anxiety phase
b. Acute disorganization phase
c. Denial phase
d. Reorganization phase



490. When preparing for an emergency bioterroism drill, the nurse instructs the drill volunteers that each biological agents requires specific patient management and medications to combat the virus, bacteria, or toxin. Which of the following statements reflect the patient management of variola virus (small pox)?

a. Small pox spreads rapidly and requires immediate isolation.
b. Acyclovir is effective against smallpox.
c. Small pox is spread by inhalation of spores.
d. Vaccination is effective only if administered within 12 to 24 hours of exposure.

491. Which of the following statements reflect the nursing management of pulmonary anthrax (B. anthracis)?

a. Prophylaxis with fluoroquinone is suggested after exposure.
b. Airborne person-to-person transmission occurs.
c. Diagnosis is by pulmonary function testing and chest x-ray.
d. Pulmonary effects include respiratory failure, shock, and death within five to seven

492. Which of the following terms refers to injuries that occur when a person is caught between objects, run over by a moving vehicle, or compressed by machinery?

a. Crush injuries
b. Blunt trauma
c. Penetrating abdominal injuries
d. Intra-abdominal injuries

493.A person suffering from carbon monoxide poisoning

a. appears intoxicated.
b. presents with severe hypertension.
c. appears hyperactive.
d. will always present with a cherry red skin coloring.


494. Treatment of an acetaminophen overdose includes the administration of

a. N-acetylcysteine (Mucomyst).
b. flumazenil (Romazicon).
c. naloxone (Narcan).
d. diazepam (Valium).

495. Which of the following statements reflect the nursing management of the patient with a white phosphorus chemical burn?

a. Do not apply water to the burn.
b. Immediately drench the skin with running water from a shower, hose or faucet.
c. Alternate applications of water and ice to the burn.
d. Wash off the chemical using warm water, then flush the skin with cool water.

496. During a disaster, the nurse sees a victim with a green triage tag. The nurse knows that the person has

a. injuries that are minor and treatment can be delayed hours to days.
b. injuries that are life-threatening but survivable with minimal intervention.
c. injuries that are significant and require medical care, but can wait hours without d. indicates injuries that are extensive and chances of survival are unlikely even with definitive care.

497. If a person has been exposed to radiation, presenting symptoms, such as nausea, vomiting, loss of appetite, diarrhea, or fatigue can be expected to occur within _______ hours after exposure?

a. 48 to 72
b. 6 to 12
c. 12 to 24
d. 24 to 48


498.Which of the following refers to a management tool for organizing personnel, facilities, equipment, and communication for any emergency situation?

a. The Incident Command System
b. Office of Emergency Management
c. National Disaster Medical System
d. The Hospital Emergency Preparedness Plan

499.Which of the following terms refers to a process by which an individual receives education about recognition of stress reactions and management strategies for handling stress?

a. Defusing
b. Debriefing
c. Follow-up
d. Critical incident stress management

500.The first step in decontamination is

a.removal of the patient's clothing and jewelry and then rinsing the patient with water.
b. a thorough soap and water wash and rinse of the patient.
c. to immediately apply personal protective equipment.
d. to immediately apply a chemical decontamination foam to the area of contamination.


ANSWERS

No comments:

Post a Comment

Categories

Amoebiasis (1) Anatomy and Physiology (42) ANATOMY AND PHYSIOLOGY Quick Review (1) ANATOMY AND PHYSIOLOGY Quick Review quiz (1) and Acid-Base Balance (3) and Dying (2) Anesthetics (2) Answers (13) antibiotics (2) antifungal (1) antiparasitics (1) Antiviral (1) Ascariasis (1) Asepsis (1) audio (2) audiobook (1) Basic Drill Answers (1) Basic Intravenous Therapy Lectures (1) Body systems (1) Bullets (1) Cancer (5) Cardiac Drugs (1) Cardiovascular (1) Cardiovascular Diseases (1) CBQ answers (1) CD A (2) CD A to Z (1) CD_A (3) CHN practice test (7) CHN practice test answers (7) Circulatory System (1) Common Board Questions (1) Common Lab Values (1) Common Laboratory tests (11) Communicable Disease Nursing (5) COMMUNICABLE DISEASES (6) Community Health Nursing (1) Comunication in Nursing (1) concepts (1) COPD (1) Coping mechanisms (1) CPR (4) Degenerative Disorders (2) Diabetes Mellitus (1) Diagnostic Procedure and tests (1) Diet (7) digestive system (1) Disorders (13) documentation and reporting (1) downloads (6) ebooks (3) Electrolyte (3) Emergency drugs (1) endocrine disorders (3) endocrine drugs (1) endocrine system (9) Endorcrine drugs (5) Family Planning (1) Fluid (3) Fluids and Electrolytes (36) FUNDAMENTALS OF NURSING (71) Gastrointestinal System (3) Git Bullets (1) GIT Disorders (5) GIT drugs (7) Grief (2) GUT (1) GUT drugs (3) handouts (1) Hematological drugs (3) Homeostasis (1) IMCI (1) immune sytem (1) increased intracranial pressure (1) Integumentary drugs (5) IV Therapy Lectures (4) Loss (2) LPN (2) LPN/LVN NCLEX (2) LRS Disorders: Infectious (4) LRS Disorders: Miscellaneous (5) Lung Cancer (4) LVN (2) maternal drill answers (7) Maternal Nursing (35) MCN (28) Medical and Surgical Nursing (61) Medical and Surgical Nursing Overview (1) Medical and Surgical Nursing Quiz (1) medications (1) MedSurg (8) MS drill answers (8) MS Drills (8) MS handouts (17) Muscular System (1) NCLEX hot topics (1) NCLEXPN (2) nervous system (1) Neuro Drugs (11) neurology (1) Neurology Anatomy and Physiology (1) NLE Practice Test (53) notes (1) NURSING (4) Nursing Bullets (3) Nursing Jurisprudence (1) Nursing Leadership and Management (1) Nursing Lectures (1) Nursing Process (1) Nursing Research (1) Nursing Research drill (1) Nursing Research drill answer (1) Nursing Slideshows (12) NURSING VIDEOS (1) Nutrition (8) Obstetric Nursing (6) OR (1) Orthopedic (1) Pain (1) Pain assessment (1) PALMER (2) Parkinson's disease (1) Pediatric Drills answers (10) Pediatrics Nursing (14) pentagon notes (2) Pericarditis (1) PHARMACOLOGY (75) Physical Assessment (11) Practice Tests (50) PRC (1) Psychiatric Nursing (18) Psychiatric Nursing Answers (7) Psychiatric Nursing Drills (7) Quizzes (5) Respiratory Disease (21) Respiratory Drugs (7) Respiratory System (3) Schizophrenia (1) self concept (1) skeletal system (1) Sleep (1) slideshow (13) stress (3) subjects (1) Surgery (1) Terms to know (1) Therapeutic Communication (1) Transcultural concepts quick review (1) Urinary System (1) video (13) Vital Signs (1)

share this blog

Share |